Peds boards (Prep questions + Medstudy) Flashcards

1
Q

Histopathologic findings of celiac disease?

A

include villous atrophy with intraepithelial lymphocytosis.

How well did you know this?
1
Not at all
2
3
4
5
Perfectly
2
Q

When should we consider Celiac test?

A

in individuals with gastrointestinal symptoms (including abdominal pain, diarrhea, and bloating) and extraintestinal symptoms (including anemia, poor growth, and dermatitis herpetiformis).

How well did you know this?
1
Not at all
2
3
4
5
Perfectly
3
Q

Initial screening for celiac disease?

A

tissue transglutaminase antibody and total IgA testing.

How well did you know this?
1
Not at all
2
3
4
5
Perfectly
4
Q

Infants with CF are at a higher risk of experiencing (eletrolyte abnormality)

A

Infants with CF are at a higher risk of experiencing hypochloremic hyponatremic dehydration than are older children and adults. Routine supplementation with sodium chloride is recommended in infants, and the dose must be increased with fever, vomiting, diarrhea, or exposure to high temperature. Older children and adults with CF in these high-risk settings should increase their consumption of salty foods to prevent dehydration.

How well did you know this?
1
Not at all
2
3
4
5
Perfectly
5
Q

Brucellosis?

A

Brucellosis is a zoonotic illness transmitted by contact with tissue or fluid of infected animals, including cattle, goats, sheep, and swine.
Consumption of unpasteurized dairy products is the top risk factor for brucellosis in children in the United States.
The symptoms of brucellosis are nonspecific and can include fever, malaise, myalgias, arthralgias, abdominal pain, and headache. Physical examination may reveal lymphadenopathy, hepatosplenomegaly, and arthritis.

How well did you know this?
1
Not at all
2
3
4
5
Perfectly
6
Q

Bartonella henselae

A
  • fever of unknown origin.
  • This zoonotic infection is typically acquired after contact with cats. However, other animals, including dogs, can be infected. Transmission can also occur via contact with infected fleas. The most common manifestation of B henselae infection is lymphadenopathy/lymphadenitis. Disseminated Bartonella infections present as fever of unknown origin; abdominal ultrasonography may reveal hepatosplenic lesions in children with disseminated infection.
How well did you know this?
1
Not at all
2
3
4
5
Perfectly
7
Q

Tularemia?

A

Francisella tularensis is the organism responsible for tularemia. Disease after a tick bite can manifest as an eschar at the bite site with local adenopathy. If inhaled, pulmonic disease can develop. Typhoidal tularemia manifests as high fever and hepatosplenomegaly.

How well did you know this?
1
Not at all
2
3
4
5
Perfectly
8
Q

H. Pylori treatment?

A

14 days of amoxicillin + PPI + clarithromycin. In case of clarithromycin resistance, add metro.

How well did you know this?
1
Not at all
2
3
4
5
Perfectly
9
Q

How to check treatment efficiency for H. Pylori?

A

Treatment effect should be assessed with a urease breath test or H pylori stool antigen test 4 weeks after completion.

How well did you know this?
1
Not at all
2
3
4
5
Perfectly
10
Q

Myringosclerosis

A
  • Myringosclerosis is characterized clinically by asymptomatic calcium and phosphate crystal deposits within the tympanic membrane that appear as whitish plaques on otoscopy.
  • Myringosclerosis is typically asymptomatic and rarely requires treatment, because hearing loss that involves the ossicles is uncommon.
How well did you know this?
1
Not at all
2
3
4
5
Perfectly
11
Q

Normal IQ?

A

Results from standardized aptitude (intelligence), achievement, and adaptive tests are typically reported as standard scores with a mean of 100 and standard deviation of 15. The normal range is considered to be within 1 standard deviation (85-115) of the mean.

How well did you know this?
1
Not at all
2
3
4
5
Perfectly
12
Q

Define intelectual disability

A

Intellectual disability is present when both intelligence and adaptive functioning are more than 2 standard deviations below normal (scores <70).

How well did you know this?
1
Not at all
2
3
4
5
Perfectly
13
Q

which complement is low on PSGN?

A

C3. C4 is usually normal

How well did you know this?
1
Not at all
2
3
4
5
Perfectly
14
Q

Treatment of PSGN?

A

supportive care, treating the sequelae of the disease (edema, hypertension, hyperkalemia, acute kidney injury), and preventing complications (encephalopathy, pulmonary edema, congestive heart failure). The initial management should include restriction of fluid and sodium intake. Hypertension and edema, resulting from sodium and fluid retention, are best treated using a potent diuretic such as furosemide, as for the boy in the vignette. If hypertension is persistent, calcium channel blockers are the preferred therapeutic agent. Severe hypertension may require intravenous infusion of nicardipine, sodium nitroprusside, or labetalol. Hyperkalemia and acute kidney injury are managed symptomatically, and they rarely require dialysis.

How well did you know this?
1
Not at all
2
3
4
5
Perfectly
15
Q

Milrinone mechanism of action?

A

Milrinone is a phosphodiesterase 3 inhibitor that increases cardiac contractility and decreases pulmonary and systemic vascular resistance, thus improving overall cardiac output.

How well did you know this?
1
Not at all
2
3
4
5
Perfectly
16
Q

Cutaneous mastocytosis

A

Cutaneous mastocytosis is characterized by macules or papules that have an orange-brown color. Stroking a lesion causes erythema and swelling (ie, Darier sign).

How well did you know this?
1
Not at all
2
3
4
5
Perfectly
17
Q

Anomalous coronary artery from the pulmonary artery (ALCAPA)?

A

Anomalous coronary artery from the pulmonary artery (ALCAPA) can present with heart failure. Prominent Q waves on electrocardiogram are a characteristic feature, given the ischemia from decreased left coronary artery blood flow as the pulmonary vascular resistance drops.

How well did you know this?
1
Not at all
2
3
4
5
Perfectly
18
Q

which intoxication let your lips cherry-red?

A

carbon monoxide poisoning

How well did you know this?
1
Not at all
2
3
4
5
Perfectly
19
Q

How to avoid kidney injury from acyclovir?

A

Intravenous isotonic hydration.

It’s caused due to precipitation of crystals in the renal tubules.

How well did you know this?
1
Not at all
2
3
4
5
Perfectly
20
Q

How many words is the kid supposed to say when he is 15 month old?

A

3-5 words at 15 months.

When the kid is 16, they usually say between 5-10 words

How well did you know this?
1
Not at all
2
3
4
5
Perfectly
21
Q

diarrhea agents transmitted by water ingestion?

A

enteric pathogens associated with untreated recreational water–associated outbreaks include norovirus, Escherichia coli, Shigella, Cryptosporidium, and others.

How well did you know this?
1
Not at all
2
3
4
5
Perfectly
22
Q

How many hours after overdose should we check the paracetamol levels in the blood?

A

4 hours

How well did you know this?
1
Not at all
2
3
4
5
Perfectly
23
Q

How does N-acetyl cysteine works during acetaminophen overdose?

A

N-acetyl cysteine (NAC), which functions by facilitating replenishment of hepatic glutathione, thus decreasing free NAPQI.

NAPQI is the responsible for liver damage.

How well did you know this?
1
Not at all
2
3
4
5
Perfectly
24
Q

describe pityriasis rosea

A

Pityriasis rosea (PR) is a self-limited papulosquamous disorder (ie, lesions are elevated and have scale). It may be infectious (human herpesvirus types 6 and 7 have been implicated) and usually occurs in the spring and fall. In as many as 80% of patients, the eruption begins with a herald patch. This lesion is round or oval, often has central clearing (although in this patient it did not), and exhibits scaling. Within 2 weeks, a more widespread eruption develops that is composed of erythematous papules and plaques. The long axes of plaques are oriented parallel to lines of skin stress. On the back, this alignment of lesions may mimic the boughs of a fir tree (ie, the “Christmas-tree” appearance)

How well did you know this?
1
Not at all
2
3
4
5
Perfectly
25
Q

special care for kids with CKD?

A

Children with CKD are at risk of experiencing abnormalities in the metabolism of calcium, phosphorus, and vitamin D and in parathyroid hormone levels. They also are at risk of developing secondary hyperparathyroidism and mineral and bone disorders, previously termed “renal osteodystrophy.” The goal of prevention of mineral and bone disorders in children with CKD includes normalizing the serum phosphorus values via dietary restriction and phosphate binders (calcium carbonate), controlling hyperparathyroidism with active vitamin D analogs (calcitriol), and supplementing vitamin D to normalize 25-hydroxyvitamin D levels.

Children with CKD may also need a potassium-restricted diet or a formula low in potassium content to prevent hyperkalemia.

How well did you know this?
1
Not at all
2
3
4
5
Perfectly
26
Q

Is protein restriction indicated for kids with CKD?

A

Protein restriction is not recommended in infants and children with CKD, because this has not been shown to improve kidney function and may impair growth.

How well did you know this?
1
Not at all
2
3
4
5
Perfectly
27
Q

Difference between IEP and 504?

A

An IEP is developed for children who qualify for special education. The IEP can provide services and supports such as specialized academic instruction, speech/language therapy, occupational therapy, and adaptive physical education. The services and supports are individualized and based on the specific student’s strengths, weaknesses, and educational needs.

Section 504 of the Rehabilitation Act of 1973 is an important law that protects the civil rights of children with physical or mental disabilities by providing these children with equal access to education in institutions that receive federal funds. This is accomplished through a 504 plan which can provide accommodations, such as changes in 1) the way material is presented (eg, visual, auditory), 2) the manner in which the child completes assignments (eg, handwritten, typed), 3) the setting in which the child is tested (eg, classroom, separate room), and 4) the time allowed for assignments or tests (eg, extended time). Children with conditions such as ADHD can qualify for a 504 plan.

How well did you know this?
1
Not at all
2
3
4
5
Perfectly
28
Q

Heritage in Duchenne dystrophy

A

Duchenne muscular dystrophy is a progressive X-linked recessive neuromuscular disorder caused by a mutation (most commonly a deletion) in the dystrophin gene encoding the dystrophin protein, which is involved in muscle membrane stability. When dystrophin is absent, the muscle membrane is less stable and becomes damaged, resulting in an inflammatory cascade with necrosis and muscle loss

How well did you know this?
1
Not at all
2
3
4
5
Perfectly
29
Q

recommendation of discharge from nicu for preterm infants?

A

The recommendation on discharge from the neonatal intensive care unit is feeding with a preterm transitional formula at 22 calories/oz with a goal of 20 to 30 g/day of weight gain after having reached a weight of 2 kg.

How well did you know this?
1
Not at all
2
3
4
5
Perfectly
30
Q

describe PLE (protein losing enteropathy)

A

Protein-losing enteropathy is defined by excessive protein loss in the intestine, which can result in hypoalbuminemia and edema. Intestinal disorders that can cause a PLE can be divided into inflammatory (resulting in increased gut protein loss), infectious, oncologic, rheumatologic, and lymphatic obstruction.

The test that would diagnose PLE is a fecal α1-antitrypsin (α1AT) level. Because α1AT is not degraded in the stomach and is not actively secreted or absorbed in the intestine, it is a reliable marker of protein loss in the intestine. Elevated fecal α1AT levels should prompt further testing for the causes of PLE.

How well did you know this?
1
Not at all
2
3
4
5
Perfectly
31
Q

Beckwith-Wiedemann syndrome (BWS)?

A

Beckwith-Wiedemann syndrome is an overgrowth disorder manifested by macrosomia, macroglossia, neonatal hypoglycemia, ear creases/pits, hemihypertrophy, and visceromegaly. Patients can also have embryonal tumors (Wilms tumor, hepatoblastoma, neuroblastoma), umbilical hernia/omphalocele, nephrocalcinosis, medullary sponge kidney disease, cardiomegaly, and nephromegaly.

How well did you know this?
1
Not at all
2
3
4
5
Perfectly
32
Q

which syndromes increase the risk of leukemia?

A

down syndrome and li fraumeni syndrome

How well did you know this?
1
Not at all
2
3
4
5
Perfectly
33
Q

risk of high exposure to doxorubicin? methotrexate? cisplatin?

A

doxorubicin - cardiomyopathy
methotrexate - hepatic injury, neurocognitive changes, and bone mineral density loss.
cisplatin - hearing loss

How well did you know this?
1
Not at all
2
3
4
5
Perfectly
34
Q

define neutropenia

A

absolute neutrophil count of less than 1,500 cells/μL.

How well did you know this?
1
Not at all
2
3
4
5
Perfectly
35
Q

most common cause of congenital hypothyroidism in the United States?

A

thyroid dysgenesis

How well did you know this?
1
Not at all
2
3
4
5
Perfectly
36
Q

define nephrotic syndrome

A

Nephrotic syndrome is characterized by proteinuria (urine protein to creatinine ratio > 2 mg/mg or urine protein > 40 mg/m2/h), hypoalbuminemia (< 2.5 gm/dL), edema, and hyperlipidemia.

How well did you know this?
1
Not at all
2
3
4
5
Perfectly
37
Q

c. perfrigens?

A

Clostridium perfringens is an anaerobic, gram-positive bacillus that commonly inhabits the human intestinal tract. The 2 most common manifestations of infection caused by C perfringens include severe soft tissue infections (eg, myonecrosis or gas gangrene) and enterotoxin-mediated food poisoning.

How well did you know this?
1
Not at all
2
3
4
5
Perfectly
38
Q

Jugular vein septic thrombophlebitis?

A

Jugular vein septic thrombophlebitis is the pathognomonic feature of Lemierre syndrome, and Fusobacterium necrophorum is the organism most commonly associated with Lemierre syndrome.

How well did you know this?
1
Not at all
2
3
4
5
Perfectly
39
Q

what is the valgus stress test? Lachman test?

A

Valgus stress test check the MCL
Lachman test check the ACL

How well did you know this?
1
Not at all
2
3
4
5
Perfectly
40
Q

Strickler Syndrome?

A

Stickler syndrome, a connective tissue disorder, is associated with midfacial hypoplasia, cleft palate, Pierre Robin sequence, hearing loss, and eye abnormalities including high-grade myopia, cataracts, and increased risk for retinal detachment.

How well did you know this?
1
Not at all
2
3
4
5
Perfectly
41
Q

Pierre-Robin sequence?

A

Pierre Robin sequence is the clinical constellation of micrognathia, cleft palate, and glossoptosis that can lead to life-threatening obstructive apnea and feeding difficulties in the neonatal period.

Pierre Robin sequence can present as an isolated finding, in association with additional congenital malformations (PRS-Plus), or in association with a defined genetic disorder (syndromic PRS). The most common genetic disorders associated with Pierre Robin sequence are 22q11.2 deletion, Treacher Collins syndrome, and Stickler syndrome.

How well did you know this?
1
Not at all
2
3
4
5
Perfectly
42
Q

A urinalysis showing granular or epithelial cell casts is suggestive of…

A

ATN

How well did you know this?
1
Not at all
2
3
4
5
Perfectly
43
Q

A urinalysis showing red blood cell casts…

A

glomerulonephritis

How well did you know this?
1
Not at all
2
3
4
5
Perfectly
44
Q

Just read…

A

In prerenal AKI, urine specific gravity is greater than 1.020, urine sodium level is less than 10 mEq/L, fractional excretion of sodium (FENa) is less than 1%, and urine to plasma osmolality is greater than 1.5. These urinary indices reflect the renal tubular reabsorption of filtered sodium as well as water in response to decreased renal perfusion. In renal AKI, the urine is dilute with specific gravity below 1.010, urine sodium level is greater than 40 mEq/L, FENa is greater than 2%, and urine to plasma osmolality is less than 1.5. The severity of AKI and renal hypoperfusion is also reflected by serum blood urea nitrogen (BUN) to creatinine ratio. A BUN to creatinine ratio of 20:1 is seen in prerenal AKI, and a ratio of 10:1 to 15:1 is seen in renal AKI.

How well did you know this?
1
Not at all
2
3
4
5
Perfectly
45
Q

How to thick the breast milk for babies with GERD?

A

xanthan gum/carob bean thickeners

How well did you know this?
1
Not at all
2
3
4
5
Perfectly
46
Q

How to thick the formula for formula fed babies with GERD?

A

rice cereal

How well did you know this?
1
Not at all
2
3
4
5
Perfectly
47
Q

What do you do if the baby continues to have GERD after formula or breastfeeding thickner?

A

If GERD continues, a 2- to 4-week trial of hydrolysate formula or elimination of cow milk in the maternal diet is recommended. If unsuccessful, referral to a pediatric gastroenterologist and/or a 4- to 8-week trial of acid suppression may be considered.

How well did you know this?
1
Not at all
2
3
4
5
Perfectly
48
Q

Describe Still / Innocent murmur

A

These murmurs are typically low-pitched, located at the lower left sternal border, and are often described as being musical in character. They may change in character or disappear with position changes, and often are not heard during the Valsalva maneuver.

How well did you know this?
1
Not at all
2
3
4
5
Perfectly
49
Q

ecthyma gangrenosum (EG) is caused by…

A

Pseudomonas aeruginosa

How well did you know this?
1
Not at all
2
3
4
5
Perfectly
50
Q

classify neutropenia

A

Neutropenia is classified as mild (1,000-1,500/µL), moderate (500-1,000/µL), severe (200-500/µL), or very severe (< 200/µL).

How well did you know this?
1
Not at all
2
3
4
5
Perfectly
51
Q

Legg Calve Perthes Disease?

A

Legg-Calvé-Perthes disease is the idiopathic interruption of blood supply to the femoral head epiphysis, with subsequent development of avascular necrosis. The blood supply is generally restored over 2 to 4 years, and the epiphysis undergoes serial fragmentation, collapse, and subsequent regeneration.

How well did you know this?
1
Not at all
2
3
4
5
Perfectly
52
Q

Management of Legg Calve Perthes disease

A

Once LCPD is identified, steps should be taken to prevent collapse of the femoral head. The patient should be provided crutches and instructed on the importance of non–weight bearing while awaiting orthopedic consultation.

How well did you know this?
1
Not at all
2
3
4
5
Perfectly
53
Q

When can we start repellents in kids?

A

The American Academy of Pediatrics states that it is safe to use DEET-containing products on children as young as 2 months of age.

How well did you know this?
1
Not at all
2
3
4
5
Perfectly
54
Q

Which repellent should we use in kids and which percentual?

A

Repellents with DEET (N,N-diethyl-3-methylbenzamide) are commonly used and are the most effective in protecting infants and children from ticks and mosquitos. The duration of protection depends on the concentration of DEET. Products containing 10% DEET typically last about 2 hours, and products with 24% DEET typically last around 5 hours. There is no known benefit in duration of protection with concentrations of DEET greater than 30% to 50%. Of the response choices, an aerosol of 30% DEET is the best option to protect a 6-month-old infant from ticks and mosquitos.

How well did you know this?
1
Not at all
2
3
4
5
Perfectly
55
Q

How to classify hypertension?

A

normal BP (< 90th percentile); elevated BP (≥ 90th percentile to < 95th percentile); stage 1 hypertension (≥ 95th percentile to <95th percentile + 12 mm Hg), and stage 2 hypertension (≥ 95th percentile + 12 mm Hg).

How well did you know this?
1
Not at all
2
3
4
5
Perfectly
56
Q

How often do you check the BP for patients who are presenting elevated BP or Hypertension?

A

Children with elevated BP are recommended to have a BP check at a second visit in 6 months. If the BP is again high, a third measurement is done in 6 months and diagnostic evaluation is initiated. For children with asymptomatic stage 1 hypertension, repeat measurement is recommended in 1 to 2 weeks. A third measurement is done in 3 months and if BP is still high, a diagnostic evaluation is initiated. Children with asymptomatic stage 2 hypertension are recommended to have repeat measurement in 1 week. Once the repeat BP reading is noted to be high, diagnostic evaluation is initiated and a referral to a subspecialist should also be considered.

How well did you know this?
1
Not at all
2
3
4
5
Perfectly
57
Q

How to investigate secondary causes for patients with hypertension?

A

The diagnostic evaluation for children with hypertension includes urinalysis, chemistry panel (including renal function), lipid profile, and renal ultrasonography. According to the current guideline, electrocardiography is not recommended to evaluate for left ventricular hypertrophy. Echocardiography is recommended to assess for left ventricular mass, geometry, function, and cardiac target organ damage.

How well did you know this?
1
Not at all
2
3
4
5
Perfectly
58
Q

Major risk factor for iron deficiency anemia in toddlers?

A

In developed countries, excessive consumption of cow milk is a major contributor to iron deficiency in toddlers, and those who consume over 32 oz daily are at highest risk.

How well did you know this?
1
Not at all
2
3
4
5
Perfectly
59
Q

Major complication of JIA?

A

A complication of systemic JIA is macrophage activation syndrome, a life-threatening illness that presents with persistent fevers, fixed rash, elevated liver enzymes, elevated triglycerides, elevated D-dimers, cytopenias, reduced fibrinogen, and decreasing erythrocyte sedimentation rate. Patients with macrophage activation syndrome can develop bleeding diathesis and cardiac, renal, or liver failure.

How well did you know this?
1
Not at all
2
3
4
5
Perfectly
60
Q

Initial treatment for JIA?

A

The mainstay of treatment for all forms of JIA with low disease activity is NSAIDs, including naproxen, indomethacin, and ibuprofen

How well did you know this?
1
Not at all
2
3
4
5
Perfectly
61
Q

Define JIA

A

Juvenile idiopathic arthritis is defined by 6 weeks’ or longer duration of chronic arthritis with no known cause occurring before age 16 years. Juvenile idiopathic arthritis is diagnosed clinically.

How well did you know this?
1
Not at all
2
3
4
5
Perfectly
62
Q

How to treat pathologic phymosis?

A

Medical treatment usually involves applying a topical steroid cream such as 0.05% betamethasone or 0.1% triamcinolone acetonide 2 or 3 times daily for 1 to 2 months along with gentle retraction. If medical therapy is unsuccessful, a urologist may be consulted. Surgery may be considered in severe cases.

How well did you know this?
1
Not at all
2
3
4
5
Perfectly
63
Q

Describe MELAS

A

MELAS (mitochondrial encephalomyopathy with lactic acidosis and stroke-like episodes) is a mitochondrial disorder that presents with childhood onset of myopathy, seizures, recurrent headaches and vomiting, sensorineural hearing loss, early signs of diabetes mellitus, stroke-like episodes associated with periods of regression, and laboratory evidence of lactic acidosis both in serum and cerebrospinal fluid.

How well did you know this?
1
Not at all
2
3
4
5
Perfectly
64
Q

Why soy formula is inappropriate for preterm babies?

A

because of the risk of osteopenia, aluminum toxicity, and prevention of adequate absorption of zinc, phosphorus, and iron

How well did you know this?
1
Not at all
2
3
4
5
Perfectly
65
Q

Primaquine should not be used for malaria prophylaxis in…

A

G6PD deficiency and pregnancy

How well did you know this?
1
Not at all
2
3
4
5
Perfectly
66
Q

Adverse effects of doxycycline?

A

gastrointestinal upset and photosensitivity

How well did you know this?
1
Not at all
2
3
4
5
Perfectly
67
Q

Drugs available for malaria prophylaxis?

A

The drugs available for malaria prophylaxis in the United States include chloroquine, mefloquine, doxycycline, atovaquone-proguanil, and primaquine. While chloroquine and mefloquine are dosed once weekly, resistance to these drugs limits the regions in which they can be used. Chloroquine resistance is widespread except in Central America west of the Panama Canal, Haiti, Dominican Republic, and most of the Middle East. Mefloquine resistance occurs in Southeast Asia.

How well did you know this?
1
Not at all
2
3
4
5
Perfectly
68
Q

Adverse effects from chloroquine?

A

gastrointestinal upset, headache, dizziness, blurred vision, pruritus, and insomnia. Chloroquine can exacerbate psoriasis.

How well did you know this?
1
Not at all
2
3
4
5
Perfectly
69
Q

Infants with congenital diaphragmatic hernia repaired in the immediate postnatal period are at risk for…

A

gastroesophageal reflux disease, growth failure, and pulmonary hypertension

How well did you know this?
1
Not at all
2
3
4
5
Perfectly
70
Q

Infants whose congenital diaphragmatic hernia was repaired with a patch are at increased risk for…

A

recurrent intestinal herniation.

Affected children should be screened with chest radiography annually until age 16 years.

How well did you know this?
1
Not at all
2
3
4
5
Perfectly
71
Q

Newborn w Delayed passage of urine and a distended urinary bladder with bilateral hydroureteronephrosis on renal ultrasonography…

A

posterior urethral valve

How well did you know this?
1
Not at all
2
3
4
5
Perfectly
72
Q

Where folate is absorbed?

A

Folate is absorbed primarily in the proximal small intestine

How well did you know this?
1
Not at all
2
3
4
5
Perfectly
73
Q

What is absorbed in the terminal ileum?

A

The terminal ileum is responsible for cobalamin and bile salt absorption

How well did you know this?
1
Not at all
2
3
4
5
Perfectly
74
Q

How to diagnose graves disease?

A

thyrotropin receptor antibodies.

How well did you know this?
1
Not at all
2
3
4
5
Perfectly
75
Q

which labs are associated with hashitoxicosis?

A

Thyroid peroxidase and antithyroglobulin antibodies are associated with Hashitoxicosis.

How well did you know this?
1
Not at all
2
3
4
5
Perfectly
76
Q

what to check if you suspect of ingestion of exogenous thyroid hormone?

A

Thyroglobulin level measurement can be helpful if ingestion of exogenous thyroid hormone is suspected.

How well did you know this?
1
Not at all
2
3
4
5
Perfectly
77
Q

Benign myoclonus of infancy

A

Benign myoclonus of infancy manifests as myoclonus, spasm with brief tonic contractions, shuddering or atonia, frequently occuring at mealtime between 3 to 8 months of age and experiencing spontaneous remission at 2 to 3 years of age.

How well did you know this?
1
Not at all
2
3
4
5
Perfectly
78
Q

Which condition is related to breath holding spells?

A

Iron deficiency anemia

How well did you know this?
1
Not at all
2
3
4
5
Perfectly
79
Q

Acute onset of hyponatremia places you in risk of…

A

Cerebral edema

How well did you know this?
1
Not at all
2
3
4
5
Perfectly
80
Q

Fast correction of hyponatremia its a risk to develop…

A

Pontine myelinolysis

How well did you know this?
1
Not at all
2
3
4
5
Perfectly
81
Q

In which condition we visualize Blue dot sign ?

A

Torsion of the appendix testis

How well did you know this?
1
Not at all
2
3
4
5
Perfectly
82
Q

Findings on US doppler in Torsion of Appendix testis

A

Findings on Doppler ultrasound, which is often performed to definitively exclude testicular torsion, reveal that testicular blood flow is normal or even increased.

How well did you know this?
1
Not at all
2
3
4
5
Perfectly
83
Q

Treatment of torsion of appendix testis?

A

rest and nonsteroidal antiinflammatory agents

How well did you know this?
1
Not at all
2
3
4
5
Perfectly
84
Q

CHARGE syndrome?

A

coloboma, heart defects, atresia of the choanae, restricted growth and development, genital anomalies, and ear anomalies and/or deafness

How well did you know this?
1
Not at all
2
3
4
5
Perfectly
85
Q

Which type of abdomen do we see in infants with diaphragmatic hernia?

A

A scaphoid abdomen

How well did you know this?
1
Not at all
2
3
4
5
Perfectly
86
Q

Persistent pulmonary hypertension of the newborn (PPHN)?

A

Persistent pulmonary hypertension of the newborn (PPHN) primarily occurs in term infants and is associated with persistent elevation in pulmonary vascular resistance, which leads to right to left shunting through the foramen ovale and ductus arteriosus and hypoxemia. Patients with PPHN present with respiratory distress—hypoxemia, tachypnea, retractions, and grunting—and a harsh systolic murmur at the left lower sternal border in the first 24 hours of life. CXR demonstrates dark, hypoperfused lung fields.

How well did you know this?
1
Not at all
2
3
4
5
Perfectly
87
Q

Findings on CXR for TTN (Transient tachypnea of newborn)?

A

Chest x-ray findings include flattened diaphragms, mild cardiomegaly, fluid in the fissures, and prominent vascular markings.

How well did you know this?
1
Not at all
2
3
4
5
Perfectly
88
Q

Corkscrew sign on Upper Gi series?

A

Malrotation

How well did you know this?
1
Not at all
2
3
4
5
Perfectly
89
Q

SMA syndrome?

A

It occurs after loss of adipose tissue anterior to the duodenum. Normally, adipose tissue separates the SMA from the third portion of the duodenum; however, when the loss of this tissue occurs, the SMA compresses the duodenum, creating a partial or complete small bowel obstruction

How well did you know this?
1
Not at all
2
3
4
5
Perfectly
90
Q

How to diagnose SMA syndrome?

A

Superior mesenteric artery syndrome is diagnosed with upper gastrointestinal series, demonstrating an obstruction in the third portion of the duodenum with contrast “to and fro” flow present.

How well did you know this?
1
Not at all
2
3
4
5
Perfectly
91
Q

Which tetanus vaccines should we give to the patient depending of their age?

A

Children younger than 7 years who require a tetanus vaccine under this management algorithm should receive the DTaP vaccine, children 7 years old and more should receive Tdap. Booster every 10 years of Td

How well did you know this?
1
Not at all
2
3
4
5
Perfectly
92
Q

Post Exposure tetanus prophylaxis?

A

Appropriate postexposure tetanus prophylaxis depends on 2 factors: if the child has received at least 3 doses of tetanus toxoid (typically from a primary series of vaccinations) and if the wound is considered to be clean or dirty. Dirty wounds include burns, bites, crush wounds, and wounds that may be contaminated with soil or dirt. If the child has previously received 3 doses of tetanus toxoid and sustained a clean wound, then no postexposure prophylaxis is indicated if the last dose of tetanus toxoid was within the previous 10 years. If the child previously received 3 doses of tetanus toxoid and sustained a dirty wound, a tetanus vaccine is indicated if the last dose was more than 5 years earlier, as described for the patient in this vignette. A child who did not have 3 previous doses of tetanus toxoid (or if it is uncertain if they did) and who sustained a clean wound requires administration of only a tetanus vaccine, while a dirty wound necessitates administration of both a tetanus vaccine and tetanus immune globulin.

How well did you know this?
1
Not at all
2
3
4
5
Perfectly
93
Q

Describe homocystinuria

A

Homocystinuria characteristically presents with a marfanoid habitus (tall stature, pectus excavatum, long limbs, and scoliosis), eye findings (severe myopia and risk of downward ectopia lentis), intellectual disability/developmental delay, and vascular thromboembolic events. Downward dislocation of the eye lens is typical rather than the upward dislocation seen in Marfan syndrome.

Vascular events are the primary cause of morbidity and mortality in CBS deficiency (also known as cystathionine β-synthase deficiency)

How well did you know this?
1
Not at all
2
3
4
5
Perfectly
94
Q

What is ALCAPA?

A

When the left coronary artery arises from the pulmonary artery and not the aorta, it leads to an ALCAPA. At birth, the condition is typically asymptomatic. The natural history of pulmonary vascular resistance is that it begins to drop in the first few months after birth. As the pulmonary vascular resistance begins to drop, and consequently, the pulmonary artery pressure begins to drop, the perfusion pressure of this coronary artery also drops, resulting in myocardial ischemia and necrosis. In some children who develop collateral vessels from the right coronary artery, this condition will manifest later

How well did you know this?
1
Not at all
2
3
4
5
Perfectly
95
Q

When Do you suspect of ALCAPA?

A

An infant with congestive heart failure and electrocardiogram showing deep Q waves in the inferior leads (II, III, AVF) should be evaluated for an anomalous left coronary artery from the pulmonary artery (ALCAPA).

How well did you know this?
1
Not at all
2
3
4
5
Perfectly
96
Q

Transient neonatal pustular melanosis - finding on microscopy?

A

Microscopic examination of pustule contents would reveal neutrophils but no organisms.

How well did you know this?
1
Not at all
2
3
4
5
Perfectly
97
Q

Erythema toxicum - finding on microscopy?

A

eosinophils

How well did you know this?
1
Not at all
2
3
4
5
Perfectly
98
Q

Patients with sickle cell disease are predispose to have infections from which organisms?

A

Children with sickle cell disease are predisposed to invasive bacterial infections from encapsulated organisms such as S pneumoniae, Haemophilus influenzae type b (Hib), and nontyphoidal Salmonella due to functional asplenia, which results in diminished antibody-mediated phagocytosis. Of these organisms, infection with S pneumoniae occurs most frequently.

How well did you know this?
1
Not at all
2
3
4
5
Perfectly
99
Q

Work up for young children with isolated horner syndrome?

A

Young children with isolated Horner syndrome should undergo careful examination for cervical and abdominal masses, measurement of urinary HVA and VMA levels, and radiologic imaging of the head, neck, and chest.

How well did you know this?
1
Not at all
2
3
4
5
Perfectly
100
Q

Why antibiotics are not indicated to treat botulism?

A

Antibiotic therapy is not indicated in the treatment of infant botulism because bacterial cell lysis and death release the intracellular toxin into the intestinal lumen; aminoglycoside agents in particular potentiate the paralytic effects of the toxin at the neuromuscular junction.

How well did you know this?
1
Not at all
2
3
4
5
Perfectly
101
Q

Why should we avoid cow’s milk until 1 year of age?

A

Due to the low bioavailability of iron in cow’s milk, limit intake to no more than 20 oz per day in children 1–5 years of age. In addition, infants should avoid cowʼs milk until 12 months of age to prevent milk protein–induced colitis resulting in occult gastrointestinal (GI) bleeding.

How well did you know this?
1
Not at all
2
3
4
5
Perfectly
102
Q

AAP Recommendation to prevent iron deficiency anemia?

A

To prevent iron deficiency, the AAP recommends iron-supplemented formula for the 1st year of life in infants who are not breastfed.

Introduce iron-rich complementary foods by 4–6 months of age. Preschoolers should consume at least 3 servings/day of iron-rich foods (e.g., iron-fortified cereals, green vegetables, lean beef, chicken).

How well did you know this?
1
Not at all
2
3
4
5
Perfectly
103
Q

Screening for iron deficiency anemia?

A

The Bright Futures/American Academy of Pediatrics (AAP) guidelines recommend routine hemoglobin testing in children 12 months of age. Anemia is defined as hemoglobin < 11.0 g/dL for both male and female children 12–35 months of age. A second screening at 15–18 months of age is recommended in children at high risk of iron deficiency anemia; those who remain at risk should again be screened at 2–5 years of age.

How well did you know this?
1
Not at all
2
3
4
5
Perfectly
104
Q

Laurence-Moon-Biedl/ Bardet-Biedl syndrome?

A

Laurence-Moon-Biedl/Bardet-Biedl syndrome is associated with retinitis pigmentosa, obesity, developmental disabilities, polydactyly, genital hypoplasia, and hypogonadism (both hyper- and hypogonadotropic hypogonadism).

How well did you know this?
1
Not at all
2
3
4
5
Perfectly
105
Q

Prader-Willi syndrome?

A

Prader-Willi syndrome is associated with hypogonadism, obesity, and intellectual disability

How well did you know this?
1
Not at all
2
3
4
5
Perfectly
106
Q

Most prevalent bacterias in AOM?

A

The most common etiologic agent of acute mastoiditis is Streptococcus pneumoniae. Other bacterial causes include Staphylococcus aureus (including methicillin-resistant S aureus), group A Streptococcus, nontypeable Haemophilus influenzae, Moraxella catarrhalis, and Streptococcus anginosus.

How well did you know this?
1
Not at all
2
3
4
5
Perfectly
107
Q

Triad of botulinum toxin?

A

The classic triad of botulinum toxin includes bulbar palsy, a descending (not ascending) paralysis, and clear sensorium.

How well did you know this?
1
Not at all
2
3
4
5
Perfectly
108
Q

DRESS ?

A

It is a rare, potentially life-threatening condition characterized by rash, hematologic abnormalities (eosinophilia), lymphadenopathy, and internal organ involvement that develops typically 2 to 6 weeks after starting the causative medication. Antiepileptic drugs, in particular carbamazepine, lamotrigine, phenytoin, and phenobarbital, are the most common cause of DRESS.

Treatment of DRESS is supportive with use of systemic corticosteroids or cyclosporine in severe cases with lung and kidney involvement.

How well did you know this?
1
Not at all
2
3
4
5
Perfectly
109
Q

Describe Stevens-Johnson syndrome

A

Stevens-Johnson syndrome is a severe drug reaction that appears up to 3 weeks after starting a medication and is characterized by mucosal lesions at 2 or more sites and widespread targetoid or macular skin lesions preceded by a prodrome of fever, sore throat, and cough

How well did you know this?
1
Not at all
2
3
4
5
Perfectly
110
Q

Describe TEN

A

Toxic epidermal necrolysis presents up to 3 weeks after starting a medication, with mucosal involvement and a morbilliform rash that rapidly blisters and exfoliates involving more than 30% of the cutaneous surface.

A positive Nikolsky sign, where the epidermis detaches with pressure from the finger, is consistent with TEN.

How well did you know this?
1
Not at all
2
3
4
5
Perfectly
111
Q

Explain refeeding syndrome

A

During starvation or malnutrition, fat and proteins are broken down, allowing gluconeogenesis to occur. Insulin levels are low, and already depleted levels of electrolytes shift to maintain homeostasis.

Once refeeding occurs, hyperglycemia results (caused by low insulin levels) and osmolar shifts occur. Intracellular shifts of magnesium and potassium occur, resulting in hypomagnesemia and hypokalemia.

Hypophosphatemia occurs due to increased utilization of phosphate for production of ATP. Increased metabolic thiamine requirements result in thiamine deficiency.

How well did you know this?
1
Not at all
2
3
4
5
Perfectly
112
Q

Which bacteria is a lancet-shaped diplococci on Gram stain with α-hemolysis on culture plate?

A

S. pneumoniae

How well did you know this?
1
Not at all
2
3
4
5
Perfectly
113
Q

When infantile hemangiomas are visualized and when do they disappear?

A

Infantile hemangiomas usually begin growing prior to 1 month of age and continue to grow until approximately 3 to 5 months. By 12 months of age involution begins and is usually complete by 4 years of age but can leave the area distorted. Between 6 and 12 months of age the proliferation and involution often occur at equal rates, leading to little change in the hemangioma during this time.

How well did you know this?
1
Not at all
2
3
4
5
Perfectly
114
Q

What is the most common cardiovascular finding in Williams syndrome?

A

The most common cardiovascular finding in Williams syndrome is supravalvar aortic stenosis

How well did you know this?
1
Not at all
2
3
4
5
Perfectly
115
Q

Which are the main characteristics of William syndrome?

A

Distinctive facies include bitemporal narrowing, periorbital fullness, stellate irides, long philtrum, upper/lower lips with a thickened vermilion, wide mouth, small jaw, large ear lobes, and a broad forehead. Individuals commonly possess a specific neurocognitive profile with overfriendliness, generalized anxiety, attention-deficit/hyperactivity disorder, strengths in verbal short-term memory and language, and weakness in visual-spatial construction. Most have intellectual disability, typically mild. Growth abnormalities are characterized by prenatal growth deficiency and slow weight gain and linear growth in the first 4 years, and a brief pubertal growth spurt resulting in short stature. Endocrine abnormalities can include idiopathic hypercalciuria and hypothyroidism.

How well did you know this?
1
Not at all
2
3
4
5
Perfectly
116
Q

Treatment for congenital CMV?

A

Antiviral treatment with oral valganciclovir for 6 months is the recommended treatment for term neonates with symptomatic congenital CMV infection with and without central nervous system disease. Therapy must be started by 4 weeks of age.

How well did you know this?
1
Not at all
2
3
4
5
Perfectly
117
Q

Which signs/symptoms do we see in patients with congenital CMV?

A

Manifestations include intrauterine growth restriction, microcephaly, intracranial calcifications, chorioretinitis, jaundice, hepatosplenomegaly, petechial/purpuric rash (because of thrombocytopenia), “blueberry muffin” lesions (because of extramedullary hematopoiesis), and elevated transaminase concentrations.

How well did you know this?
1
Not at all
2
3
4
5
Perfectly
118
Q

What’s the treatment for labial adhesions?

A

Nearly all children with labial adhesions will have complete self-resolution within 18 months, therefore asymptomatic individuals should receive reassurance. Symptomatic individuals (those with urinary problems such as urinary tract infections or urinary retention) should first undergo a trial of twice daily topical estrogen cream. Adverse effects of estrogen cream, such as vulvar hyperpigmentation or breast budding, will reverse once the cream is discontinued. If adverse effects become unacceptable or if adhesions are still present after 8 weeks of topical treatment, manual separation in the office may be performed under local anesthetic with postprocedural care involving daily application of a lubricant for several months to prevent reformation of adhesions. Referral to a specialist for surgical lysis may be necessary for patients who are unresponsive to conventional treatments.

How well did you know this?
1
Not at all
2
3
4
5
Perfectly
119
Q

Describe the cause for breastmilk jaundice

A

Breastmilk jaundice occurs most commonly in breastfed infants and is secondary to elevated levels of β-glucuronidase in breast milk. β-glucuronidase unconjugates bilirubin from glucuronic acid and makes it available for reabsorption. Patients develop jaundice that peaks at 2 weeks of life. Usually, it does not require any treatment.

How well did you know this?
1
Not at all
2
3
4
5
Perfectly
120
Q

Breastfeeding jaundice?

A

Patients with breastfeeding jaundice present in the 1st week of life with jaundice, dehydration, and weight loss caused by hypovolemia. These infants develop indirect hyperbilirubinemia secondary to decreased numbers of bowel movements and increased enterohepatic circulation of bilirubin.

How well did you know this?
1
Not at all
2
3
4
5
Perfectly
121
Q

At which age the testes descend to the scrotum?

A

In males at 34 weeks of gestation, the testes are usually palpable only in the inguinal canal, descending into the upper scrotum by 36–39 weeks.

How well did you know this?
1
Not at all
2
3
4
5
Perfectly
122
Q

Pentalogy of Cantrell?

A

Pentalogy of Cantrell includes omphalocele and CDH.

How well did you know this?
1
Not at all
2
3
4
5
Perfectly
123
Q

Which class of anti-hypertensive are related to congenital malformations?

A

ACEIs

How well did you know this?
1
Not at all
2
3
4
5
Perfectly
124
Q

Which congenital malformations are expected when the patient take ACEIs during pregnancy?

A

The most common congenital anomaly associated with use of an ACEI during the 1st trimester is malformation of the cardiovascular and central nervous systems. Exposure to ACEIs during the 2nd and 3rd trimester has also been associated with serious adverse fetal defects, especially in the kidneys. Pregnancy is often complicated by oligohydramnios due to a decrease in the production of urine, which may be associated with intrauterine demise. Surviving infants often have limb and hand anomalies, widely separated eyes, low-set ears, and facial deformities resulting from compression of the fetus within the uterine cavity due to oligohydramnios. ACEIs cross the placenta and inhibit the renin-angiotensin system, causing a decrease in the glomerular filtration rate and urine production.

How well did you know this?
1
Not at all
2
3
4
5
Perfectly
125
Q

Genetic syndrome associated with hyperinsulinemia?

A

Beckwith-Wiedemann

How well did you know this?
1
Not at all
2
3
4
5
Perfectly
126
Q

Critical labs for hypoglycemia?

A

Guidelines recommend obtaining a “critical sample” of blood at the time of hypoglycemia (glucose <50 mg/dL to aid in determining the etiology of persistent neonatal hypoglycemia. In addition to confirming the plasma glucose level, measurements of insulin, β-hydroxybutyrate (a ketone body), free fatty acids, growth hormone, and cortisol levels are useful tests to perform.

How well did you know this?
1
Not at all
2
3
4
5
Perfectly
127
Q

Reiter syndrome?

A

arthritis, conjunctivitis, and urethritis

How well did you know this?
1
Not at all
2
3
4
5
Perfectly
128
Q

Whats the treatment for reactive arthritis?

A

Management is primarily supportive with nonsteroidal anti-inflammatory medications, such as ibuprofen, cold packs to the affected joints, and rest. Prednisone is not used to treat this condition unless it becomes chronic. Antibiotics such as amoxicillin or ceftriaxone are used only if indicated for the underlying infection.

How well did you know this?
1
Not at all
2
3
4
5
Perfectly
129
Q

Most appropriate treatment for common warts in kids?

A

Although a variety of treatments for common warts are available, for most children keratolytic therapy using topical salicylic acid is the preferred initial approach.

How well did you know this?
1
Not at all
2
3
4
5
Perfectly
130
Q

Most common analgesia given during circumcision?

A

A subcutaneous ring block or dorsal penile block should be used during circumcision.

How well did you know this?
1
Not at all
2
3
4
5
Perfectly
131
Q

Classify the different types of IVH

A

Grade 1 IVH: Bleeding within the germinal matrix
Grade 2 IVH: Blood noted in the ventricle
Grade 3 IVH: Blood noted in the ventricle with ventricular dilation
Grade 4 IVH: Blood extends into the brain parenchyma

How well did you know this?
1
Not at all
2
3
4
5
Perfectly
132
Q

Describe achondroplasia

A

Achondroplasia is a skeletal dysplasia caused by mutations in the fibroblast growth factor receptor 3 gene (FGFR3) and is inherited in an autosomal dominant fashion. It is the most common type of skeletal dysplasia, a group of conditions that affect bone development. Children with achondroplasia have disproportionate short stature, long bone shortening that is most prominent in the humerus and femur, brachydactyly (short fingers and toes), kyphoscoliosis, lumbar lordosis, and macrocephaly. Early gross motor delays typically exist because of atypical physical proportions; delays resolve by age 2 to 3 years. Children with achondroplasia are at increased risk of developing otitis media, obstructive sleep apnea, obesity, chronic joint pain, and cervical medullary compression.

How well did you know this?
1
Not at all
2
3
4
5
Perfectly
133
Q

Describe Osteogenesis Imperfecta

A

Osteogenesis imperfecta is a heterogeneous bone matrix disorder characterized by increased bone fragility. Although there are variable phenotypic presentations, all children with osteogenesis imperfecta have bone fragility, osteoporosis, bone deformities, and multiple or atypical fractures. Some also have blue sclera, hearing loss, dental abnormalities, increased ligamentous and skin laxity, and/or easy bruising. Several gene mutations have been identified, but the most common are autosomal dominant mutations in COL1A1 or COL1A2 which encode chains of type 1 collagen.

How well did you know this?
1
Not at all
2
3
4
5
Perfectly
134
Q

Describe arthrogryposis

A

Arthrogryposis (also known as “arthrogryposis multiplex congenita”) is a condition in which multiple joint contractures develop before birth. It can be associated with multiple diagnoses that decrease fetal movement in utero, including central nervous system disorders, neuromuscular diseases, connective tissue or muscular disorders, and trisomies.

How well did you know this?
1
Not at all
2
3
4
5
Perfectly
135
Q

How many kcal/day do a preterm baby require ?

A

premature infants typically require 105 to 130 kcal/kg per day, with a weight gain goal of 15 to 20 g/day.

How well did you know this?
1
Not at all
2
3
4
5
Perfectly
136
Q

First diagnostic test on CAH?

A

A 17-hydroxyprogesterone level is the first-line diagnostic test for congenital adrenal hyperplasia arising from 21-hydroxylase deficiency.

How well did you know this?
1
Not at all
2
3
4
5
Perfectly
137
Q

For which of the following conditions would balloon atrial septostomy be helpful?

A

Transposition of great arteries, tricuspid or pulmonary atresia.

How well did you know this?
1
Not at all
2
3
4
5
Perfectly
138
Q

Black widow spider bite symptoms?

A

Severe cramping, which may occur as soon as 10–15 minutes after the bite of a black widow, is characteristic of envenomation. The neurotoxic venom can also cause diaphoresis, emesis, chest and abdominal pain, agitation, and hypertension. Treatment consists of muscle relaxants and medication to control pain. IV calcium has been shown to not be effective. An antivenin exists but is not often used due to the low mortality of black widow envenomations.

How well did you know this?
1
Not at all
2
3
4
5
Perfectly
139
Q

How are the Phosphorus level on vitamin D deficiency?

A

Vitamin D deficiency may result in hypocalcemia. However, phosphorus levels are typically low in vitamin D deficiency. Parathyroid hormone levels are high in response to low calcium levels, resulting in excess renal phosphorus excretion through the kidney.

How well did you know this?
1
Not at all
2
3
4
5
Perfectly
140
Q

Most common cause of tinea capitis and how is it treated?

A

Trichophyton is the most common cause of tinea capitis, followed by Microsporum species.
Tinea capitis is treated with a systemic oral antifungal agent such as griseofulvin, terbinafine, or fluconazole.

How well did you know this?
1
Not at all
2
3
4
5
Perfectly
141
Q

Which are the indications of VCUG after UTI episode?

A

A voiding cystourethrogram (VCUG) should be performed in children of any age with: ≥ 2 febrile UTIs; an abnormal renal and bladder ultrasound following a first febrile UTI; a UTI associated with a temperature of ≥ 102.2°F (39°C) and a pathogen other than E. coli; failure to thrive; or hypertension.

How well did you know this?
1
Not at all
2
3
4
5
Perfectly
142
Q

Which labs are elevated in the setting of streptococcus infection on the skin / pharyngitis?

A

Elevated DNase B antibody is associated with cutaneous streptococcal infection, while elevated antistreptolysin O levels are present following streptococcal pharyngitis.

How well did you know this?
1
Not at all
2
3
4
5
Perfectly
143
Q

Scabies treatment?

A

First-line therapy for those > 2 months of age is permethrin 5% cream, applied from the neck down and rinsed after 8–14 hours. In infants and young children, apply scabicide cream/lotion to the entire head/scalp and neck along with the remainder of the body. Lindane 1% lotion is generally not recommended, especially in infants, due to the potential for central nervous system toxicity.

144
Q

In which conditions do we see basophilic stippling ?

A

Basophilic stippling is characterized by the presence of bluish granules of various sizes scattered throughout the cytoplasm of red blood cells. Although not diagnostic of lead poisoning, it is a common finding in this disorder. Basophilic stippling may also be seen in patients with thalassemia, chronic alcohol use, heavy metal poisoning, sickle cell anemia, megaloblastic anemia, and sideroblastic anemia.

145
Q

Chronic Constipation management?

A

First-line therapy for functional constipation is daily polyethylene glycol; this should continue for at least 2 months and should not stop unless symptoms have resolved for at least 1 month.

146
Q

Cardiovascular issues on Marfan syndrome?

A

Cardiovascular features, which are the most common cause of morbidity and mortality, include aortic dilatation with risk for tear and rupture, mitral valve prolapse and regurgitation, aortic valve insufficiency, tricuspid valve prolapse, and enlargement of the proximal pulmonary artery.

147
Q

Risk factor for preseptal cellulitis?

A

Preseptal cellulitis is associated with sinusitis, dental infections, trauma to the skin (eg, insect bite, atopic dermatitis, impetigo), and ocular conditions such as hordeolum or dacryocystitis.

148
Q

What are the malformations expected in a neonate who was born from a mom with PKU?

A

Neonates born to mothers with phenylketonuria (PKU) are at risk for complications related to maternal phenylalanine levels. Neonates born to mothers with PKU are exposed to elevated phenylalanine levels and may develop intrauterine growth restriction, microcephaly, structural cardiac anomalies, and developmental delays. The risk of developing these complications is proportional to maternal serum phenylalanine levels.

149
Q

scrotal pain alleviated with elevation…

A

epididymitis

150
Q

Indication for each hearing devices

A

Behind-the-ear hearing aids may be provided for infants as young as age 2 months and are the first-line intervention for infants with hearing loss. Bone-anchored hearing aids are appropriate for children at least 5 years old with conductive hearing loss. Brainstem implantation in children is considered only in special situations. Cochlear implants may be considered if hearing aids are not effective; they are approved by the US Food and Drug Administration for children as young as 12 months of age.

151
Q

Sensorineural hearing loss can result from exposure to which medications?

A

aminoglycosides, loop diuretics, and cisplatin

152
Q

Management of caustic product ingestion?

A

Initial management of ingestion of a caustic product is observation. Upper endoscopy is recommended 12–24 hours after ingestion. Doing the endoscopy before 12 hours may not show the full extent of the injury. If the child has developed upper airway disease, you will likely need to intubate.

153
Q

Alagille Syndrome - main features?

A

The physical features of the pointed chin, butterfly vertebrae, and heart murmur in conjunction with jaundice are consistent with Alagille syndrome.

154
Q

Findings on bronchiolitis chest Xray?

A

Chest radiography is not recommended in cases of bronchiolitis. The typical findings of hyperinflation, peribronchial thickening, and patchy opacification likely reflect atelectasis and viral pneumonitis, which are not indications for antibiotic use.

155
Q

What is the Leading cause of sensorineural hearing loss in United States?

A

Cytomegalovirus

156
Q

Describe IgA bullous dermatosis

A

IgA bullous dermatosis is an idiopathic or drug-induced subepidermal blistering disorder. A small subset of patients have this reaction in association with vancomycin and other antibiotics. The clinical appearance of linear IgA bullous dermatosis is similar to that of erythema multiforme. Direct immunofluorescence is required to confirm the diagnosis and is characterized by linear IgA deposition at the dermal-epidermal junction. The lesions of linear IgA dermatosis have a predilection for the face, lower trunk, and genitalia. Rings of blisters—the “string of pearls sign”—are common on the trunk.

157
Q

Transverse myelitis symptoms?

A

Symptoms typically develop rapidly and include localized lower back pain; sudden paresthesias, often described as painful burning, pricking, or tingling sensations in the legs; loss of sensation; and partial or even complete paralysis of the legs. Patients are typically very sensitive to touch. Urinary bladder and bowel incontinence is common. Respiratory symptoms may also occur.

158
Q

How is the MRI on transverse myelitis? What about the CSF?

A

MRI typically reveals fusiform swelling of 1 or more cord segments. Cerebrospinal fluid is abnormal in up to 50% of patients, with moderate elevation of protein and lymphocytes (elevated protein level and moderate lymphocytosis). Glucose levels are normal.

159
Q

How are the CSF protein levels on pseudotumor cerebri?

A

Decreased levels of protein have been described in pseudotumor cerebri

160
Q

Langerhans cells histiocytosis?

A

Following biopsy, electron microscopy reveals characteristic tennis racket–shaped Birbeck granules throughout the cytoplasm of the Langerhans cell. The majority (80%) of patients with LCH have bony lesions, most commonly in the skull, femur, humerus, and vertebrae.

Mastoid bone lesions are often associated with recurrent ear infections and chronically draining ears. Cutaneous lesions occur in up to 50% of patients. Papular, erythematous, Candida-like lesions may occur in the groin, abdomen, and/or trunk. Seborrheic-like lesions are also common. Other skin lesions may be pustular, purpuric, petechial, or vesicular. Additional clinical findings may include localized or widespread lymph node enlargement and hepatomegaly. Pituitary dysfunction associated with diabetes insipidus may result in the inability to concentrate the urine, causing decreased urine osmolarity; therefore, decreased urine osmolarity is most likely to be identified.

161
Q

Which cardiac condition is present in neonatal lupus?

A

The most common cardiac conduction abnormality in infants with NSLE is complete congenital heart block (CHB) or third-degree atrioventricular block

162
Q

Clinical diagnosis of NF1?

A

The clinical diagnosis of NF1 is made when at least 2 of the following features are present:

Café au lait macules (CALs) (at least 6; larger than 5 mm prepubertal, larger than 15 mm postpubertal)
Axillary and/or inguinal freckling
Cutaneous neurofibromas (at least 2)
Iris Lisch nodules (at least 2)
Plexiform neurofibroma (at least 1)
Optic glioma
Osseous lesion (sphenoid dysplasia or tibial pseudoarthrosis)
First-degree relative with NF1

163
Q

What are the contraindications of rotavirus vaccine?

A

Rotavirus vaccination is contraindicated in children with a history of intussusception, severe combined immunodeficiency (SCID), or a severe allergic reaction to the vaccine or one of its components.

164
Q

How much is the MCV in macrocytic anemia?

A

> 80

165
Q

Mention two stool inflammatory markers

A

lactoferrin and calprotectin

166
Q

What’s the treatment for nasal polyps?

A

Treatment options for nasal polyps include intranasal corticosteroids, such as fluticasone nasal spray, nasal saline rinses, or systemic corticosteroids. If symptoms persist despite medical treatment, polyps can be surgically removed.

167
Q

Describe Acute Interstitial nephritis

A

Acute interstitial nephritis is usually caused by an allergic reaction to a drug, and is characterized by fever, rash, and eosinophilia.

168
Q

What’s the triad on hemolytic uremic syndrome?

A

Those with hemolytic uremic syndrome may have a history of preceding diarrhea and the classic triad of hemolytic anemia, renal failure, and thrombocytopenia.

169
Q

clinical presentation of benign Rolandic epilepsy?

A

the clinical presentation of seizure is typically described as facial and arm twitching, speech arrest with accompanying oropharyngeal guttural sounds, and hypersalivation with preservation of consciousness. Generalized tonic-clonic seizures may occur in a subset of patients. Seizures spontaneously remit in adolescence, usually by 16 years of age.

170
Q

How is the EEG on benign Rolandic epilepsy?

A

Electroencephalography in benign rolandic epilepsy demonstrates biphasic independently bilateral centrotemporal spikes on a normal background, often identified in light sleep.

171
Q

Which anti-seizure meds do not interfere with contraceptive pills?

A

Enzyme induction does not occur in patients treated with levetiracetam, ethosuximide, gabapentin, zonisamide, or valproate (a.k.a. valproic acid)

172
Q

Most common cause of in-toeing?

A

The most common cause of in-toeing is internal tibial torsion, which presents in the 2nd year of life when the child starts to walk.

173
Q

Metatarsus adducts?

A

Metatarsus adductus causes in-toeing noted at birth and occurs from intrauterine crowding. It typically resolves spontaneously within the 1st year of life. In metatarsus adductus, the forefoot is adducted and can be flexible or rigid.

174
Q

Define status epilepticus

A

Status epilepticus is defined as repeated seizures without regaining consciousness or a seizure prolonged for at least 5 minutes.

175
Q

Antibiotic prophylaxis for UTI? Indication and which one is the first choice.

A

Antibiotic prophylaxis is indicated for children with low-grade VUR with recurrent UTI, grade III or higher VUR, or non–toilet-trained children with any grade of VUR. Prophylactic antibiotics sterilize the urine and decrease the risk of recurrent UTI. The antibiotic is preferably given once daily at bedtime. Prophylactic antibiotic dosing is generally one-fourth to one-half of the daily treatment dose. Commonly used agents include TMP-SMX, trimethoprim alone, or nitrofurantoin. Trimethoprim-sulfamethoxazole is preferred for prophylaxis because of its bactericidal action.

Amoxicillin prophylaxis is recommended for infants younger than 2 months, because both TMP/SMX and nitrofurantoin interfere with hepatic conjugation and increase the risk of hyperbilirubinemia. Cefixime, a third-generation cephalosporin, is used to treat UTI but not for prophylaxis. Nitrofurantoin is bacteriostatic and indicated as prophylaxis for children with sulfa allergy and those having breakthrough infection with TMP/SMX. Cephalexin is used for children with breakthrough infections with TMP/SMX or nitrofurantoin prophylaxis. Cephalexin is not the preferred choice for UTI prophylaxis because of the risk of developing resistance.

176
Q

Which antibiotic should be chosen in animal bite?

A

Amoxicillin-clavulanate is the drug of choice for treatment of dog, cat, or other mammal bites. For children with penicillin allergy, oral alternatives are extended-spectrum cephalosporin plus clindamycin or trimethoprim-sulfamethoxazole plus clindamycin.

177
Q

Most common cause of in-toeing in children older than 3yo?

A

Femoral anteversion is the most common cause of in-toeing in children older than 3 years

178
Q

Most common cause of in-toeing in children younger than 3yo?

A

internal tibial torsion

179
Q

Common diet deficiency find in infants who drink goat milk?

A

Folate deficiency is a well-known consequence of a goat-milk diet in infants.

180
Q

Describe the features of fetal hydantoin syndrome (FHS), or phenytoin embryopathy

A

Associated head and neck abnormalities may include microcephaly, brachycephaly, midfacial hypoplasia, low-set ears, mild hypertelorism, depressed nasal bridge, and cleft lip and palate. The digits of the hands and feet are often hypoplastic or even absent. Coarse hair and hirsutism are common.

181
Q

Kallman Syndrome?

A

Patients with secondary hypogonadism associated with 1 or more nongonadal congenital abnormalities often have Kallmann syndrome. Characterized by anosmia or hyposmia detected during testing of cranial nerve 1 (olfactory nerve), patients may also have red-green color blindness, midline facial abnormalities (such as cleft lip/palate), urogenital tract abnormalities, neurosensory hearing loss, and mirror movements.

182
Q

What’s the frequency of BM in exclusively breastfed infants?

A

Exclusively breastfed infants often stool only once every 3 to 4 days.

183
Q

When should we consider antibiotics to treat C. jejuni and which one is indicated?

A

Antibiotics can be considered for a child with Campylobacter gastroenteritis with high fever, bloody diarrhea, worsening symptoms, symptoms lasting longer than 7 days, or a compromised immune system. Antibiotic treatment shortens the duration of illness by 1 to 3 days. When indicated, the antibiotic treatment choices include azithromycin for 1 to 3 days and erythromycin for 5 days.

184
Q

What’s the treatment for rosacea?

A

topical metronidazole is often effective in the treatment of mild-to-moderate cases.

Alternative topical therapies include azelaic acid, sulfacetamide-sulfur, benzoyl peroxide with or without a topical antibiotic, and retinoids.

185
Q

Describe vocal cord dysfunction

A

VCD can be difficult to distinguish from asthma. Patients with VCD tend to present with inspiratory stridor, throat tightness, and complaints of difficulty “taking a breath” that are worst during maximal exercise and resolve within the first 5 minutes of stopping the activity.

These patients show no response to short-acting β-agonist therapy. Adduction of the vocal cords, visualized using direct laryngoscopy or flattening of the inspiratory loop on spirometry is diagnostic; however, in the absence of symptoms, these tests are likely to be normal.

1st line therapy involves learning breathing techniques, usually in consultation with a speech language pathologist.

186
Q

Which antihypertensive medication should we avoid if the patient has sulfa allergy?

A

Furosemide

187
Q

Contraindications of MMR vaccine?

A

MMR vaccination is contraindicated in patients with a history of anaphylaxis to MMR vaccination; neomycin or gelatin; pregnancy; and severe immunodeficiency. Those taking prolonged systemic corticosteroid are also considered immunocompromised; MMR vaccination is generally contraindicated in children receiving high-dose (2 mg/kg/day or ≥ 20 mg/day) oral corticosteroids for ≥ 14 days.

188
Q

What’s the treatment for catch scratch disease?

A

Azithromycin may hasten resolution of lymphadenopathy in the initial stages of the disease. Suppurative nodes may be drained by needle aspiration or complete excision (avoid I&D). Systemic disease is often treated with a combination of rifampin + gentamicin or rifampin + azithromycin.

189
Q

Describe papular acrodermatitis of childhood

A

Papular acrodermatitis of childhood (a.k.a. Gianotti-Crosti syndrome) is characterized by multiple slightly edematous and erythematous papules distributed on the face, buttocks, and extensor surface of the forearms and legs while sparing the trunk. The papules (or, less commonly, papulovesicles) may persist for weeks to months and are typically nonumbilicated, monomorphous, and brownish-tan in color, ranging in size from 1 mm to 10 mm. The Koebner phenomenon (lesions that appear or become more prominent at sites of trauma) is often demonstrated. Upper respiratory symptoms, fever, diarrhea, hepatosplenomegaly, and/or lymphadenopathy may be noted prior to the appearance of the rash,

190
Q

What’s the clinical diagnosis of NF type 2?

A

The clinical diagnosis of NF2 is based upon the presence of:

Bilateral vestibular schwannomas
Unilateral vestibular schwannoma accompanied by 2 of the following: meningioma, schwannoma, neurofibroma, glioma, cataract in the form of subcapsular lenticular opacities, or cortical wedge cataract

191
Q

Congenital varicella features?

A

Congenital varicella syndrome is characterized by severe abnormalities involving the central nervous system (microcephaly, cortical atrophy, seizures), eyes (chorioretinitis), skin (scarred skin lesions along a dermatome), and skeleton (limb hypoplasia).

192
Q

When do we need to give immune globulin (VZIG) for babies in the setting of maternal varicella?

A

Neonatal varicella occurs when the mother develops infection around the time of delivery (perinatal transmission), with the risk to the neonate being greatest when maternal varicella infection occurs 5 days before through 2 days after delivery.

193
Q

Indications for tympanostomy tube insertion…

A
  • Bilateral OME that lasts 3 months or longer with conductive hearing loss
  • Bilateral or unilateral OME that lasts at least 3 months together with risk factors for speech, language, or learning problems (eg, craniofacial anomalies, neurodevelopmental disabilities, etc)
  • Bilateral or unilateral OME that lasts 3 months or longer with tympanic membrane or middle ear damage
  • Recurrent acute otitis media (AOM) with OME at the time of evaluation
194
Q

Which anti depressives are FDS approved in pediatrics?

A

The FDA has approved 2 medications for treatment of depression in adolescents—fluoxetine and escitalopram.

195
Q

Psoriasis is associated with an increased risk for…

A

Psoriasis is associated with an increased risk for obesity, metabolic syndrome, arthritis, psychiatric disorders including anxiety and depression, and less commonly, Crohn disease, uveitis, and rheumatoid arthritis.

196
Q

When should we screen infants with perinatal exposure for hepatitis C?

A

American Academy of Pediatrics guidelines recommend serologic testing of infants with perinatal exposure to hepatitis C virus at 18 months of age.

197
Q

Tell me the association between the CO2 and pH on the BG

A

For every 10 mm Hg increase in Pco2, the pH is expected to decrease by ~0.08 units

For every 10 mm Hg decrease in Pco2, the pH is expected to increase by ~0.08 units

198
Q

What’s the most common test used to diagnose pancreatic insufficiency ?

A

fecal elastase-1 level.

199
Q

Sturge Weber Syndrome?

A

Sturge-Weber syndrome is diagnosed by the presence of at least 2 of the following: facial port wine stain, leptomeningeal angiomatosis, and elevated intraocular pressure.

200
Q

Darier sign?

A

A great majority of patients with cutaneous mastocytosis develop localized erythema and urticaria after gentle rubbing or stroking of a lesion (Darier sign).

201
Q

Nikolsky sign?

A

Removal of the upper layer of the epidermis after stroking the lesion describes Nikolsky sign, which is often observed in patients with staphylococcal scalded skin syndrome or toxic shock syndrome.

202
Q

Auspitz sign?

A

Fine punctate bleeding points following removal of an overlying scale describes Auspitz sign, which is typically seen in patients with psoriasis.

203
Q

Which condition will give schistocytes on peripheral blood smear?

A

Hemolytic Uremic Syndrome (HUS)

204
Q

Describe eczema coxsackium

A

Eczema coxsackium is an atypical skin rash in children with atopic dermatitis characterized by vesicles and erosions within areas of eczema.

205
Q

Which testicle size indicates the onset of puberty?

A

Testicular enlargement of 4 mL in volume indicates the onset of puberty.

206
Q

What are the risk factors for cardiorespiratory events in a car seat?

A

Risk factors for cardiorespiratory events while restrained in a semi-reclined car seat include prematurity, low birth weight, hypotonia (eg, Down syndrome), micrognathia (eg, Pierre Robin sequence), and congenital heart disease.

207
Q

What is the Apt-Downey test?

A

For neonates and infants (up to age 8 weeks) with hematemesis, the Apt-Downey test differentiates whether the source of the blood is the mother or the neonate/infant.

208
Q

Which vaccines are adjusted for preterm neonates?

A

The vaccination schedule for preterm neonates and infants should be adjusted for 2 vaccines, rotavirus and hepatitis B. The first dose of live rotavirus vaccine, usually given at 2 months of age, should not be given while the infant is still hospitalized because of the risk of transmission to other high-risk infants. This dose should be given when the infant is to be discharged and between the ages of 6 weeks and 14 6/7 weeks. If the infant requires rehospitalization within 2 to 3 weeks of receiving the vaccine, physicians may consider placing the infant under contact precautions.

209
Q

At what time to give a hepatitis B vaccine to a neonate?

A

Preterm baby with positive perinatal exposure: the hepatitis B vaccine should be given within 12 hours of birth to protect against perinatal transmission, because his mother’s hepatitis B surface antigen is positive.

Administration of hepatitis B vaccine at 24 hours of age is appropriate for infants born weighing at least 2 kg whose mothers test negative for hepatitis B surface antigen.

Hepatitis B vaccine administration at 1 month of age, or at the time of hospital discharge, is appropriate for infants born weighing less than 2 kg whose mothers have a negative hepatitis B surface antigen.

210
Q

Craniopharyngioma?

A

Craniopharyngioma is the most common suprasellar tumor seen in children. It is a benign neoplasm originating from the remnants of the Rathke pouch that can cause significant problems due to its location and mass effect. Common presenting symptoms are those of increased intracranial pressure (eg, headache, nausea, vomiting), mass effect on the optic chiasm (eg, vision problems, visual field deficits), and pituitary gland dysfunction. On neuroimaging, cysts and calcifications are often present, with the latter better visualized on computed tomography scan. Treatment is associated with significant morbidity, specifically panhypopituitarism, diabetes insipidus, hypothalamic hyperphagia, and vision loss.

Many children have at least 1 pituitary hormone deficiency at the time of diagnosis of craniopharyngioma, and the majority have multiple pituitary hormone deficiencies after resection. Growth hormone is the most common pituitary hormone deficiency at the time of diagnosis followed by gonadotropin deficiency, then deficiencies of antidiuretic hormone, thyroid-stimulating hormone (TSH), and adrenocorticotropic hormone (ACTH). Growth problems (most commonly), pubertal delay or arrest, diabetes insipidus, and weight gain may be present at the time of diagnosis.

211
Q

Differentiate hyponatremia in primary polydipsia and in SIADH

A

Hyponatremia, a low serum osmolality, and low urine osmolality are consistent with primary polydipsia. Hyponatremia, a low serum osmolality, and high urine osmolality are consistent with the syndrome of inappropriate diuretic hormone.

212
Q

What do you expect to see in diabetes insipidus?

A

Hypernatremia, a high serum osmolality, and low urine osmolality are consistent with diabetes insipidus. Other laboratory findings consistent with dehydration may be seen with diabetes insipidus. The diagnosis is made if the serum osmolality is >300 mOsm/kg when the urine osmolality is <300 mOsm/kg. A water deprivation test may be required to make the diagnosis. Diabetes insipidus can be differentiated from other causes of hypernatremic dehydration by the presence of dilute urine (ie, low urine osmolality). Hypernatremia, high serum osmolality, and high urine osmolality (ie, concentrated urine) are consistent with hypernatremic dehydration with a normal antidiuretic hormone response.

213
Q

Which finding can we see in Beckwith-Wiedemann syndrome?

A

Beckwith–Wiedemann syndrome is an overgrowth disorder characterized by macrosomia, macroglossia, body asymmetry, abdominal wall defects, and increased risk for embryonal tumors. Recommended tumor screening for children diagnosed with Beckwith–Wiedemann syndrome includes a serum α-fetoprotein level (the tumor marker for hepatoblastoma) every 3 to 4 months until age 4 years, and abdominal ultrasonography (looking for embryonal tumors) every 3 to 4 months until age 8 years.

214
Q

Describe the major findings in tuberous sclerosis

A

Skin:
Hypomelanotic macules (≥3 that are >5 mm)
* Facial angiofibromas (≥3), shagreen patch
* Fibrous cephalic plaque
* Ungual fibroma (≥2)

Brain:
Subependymal nodules
* Cortical dysplasia, seizures
* Subependymal giant cell astrocytoma (SEGA)

Kidney: Angiomyolipomas (AML)

Heart: Rhabdomyoma

Lung: Lymphangioleiomyomatosis (LAM)

Eyes: Multiple retinal nodular hamartomas

215
Q

Long term effect of etoposide?

A

The long-term adverse effects of etoposide, a topoisomerase II inhibitor, include acute myeloid leukemia (AML) and myelodysplastic syndrome.

216
Q

Long term effect of cyclophosphamide and cytarabine?

A

Cyclophosphamide and cytarabine are alkylating agents with delayed effects including gonadal failure, bladder cancer, and treatment-related AML and myelodysplastic syndrome.

217
Q

Steps to declare brain death

A

To declare brain death the physician must document:

  1. Absent cortical function: No spontaneous movement or response to voice or painful stimuli
  2. Absent brainstem function: Absent pupillary response with fixed and dilated pupils, absent corneal reflex, cough or gag, and absent oculocephalic (doll’s eyes) and oculovestibular (cold calorics) reflexes
  3. Apnea test: No spontaneous respiratory effort with a documented Pco2 above 60 mm Hg and greater than 20 mm Hg change above baseline Pco2
218
Q

How many brain death examination should the patient have?

A

For brain death to be determined in children, 2 separate examinations by different physicians are required, with an interval of 24 hours for term newborns (>37 weeks’ gestational age) up to age 30 days, and 12 hours for children who are between 30 days and 18 years of age.

219
Q

calculation for anion gap?

A

Anion Gap = (Na+ + K+) – (Cl- + HCO3-)

220
Q

Acute treatment for anaphylaxis?

A

The treatment for acute anaphylaxis is 0.01 mg/kg of 1:1,000 concentration of epinephrine. The correct dose of auto-injector epinephrine is 0.15 mg for children less than 30 kg and 0.3 mg for children greater than or equal to 30 kg. The preferred route is intramuscular.

221
Q

Describe granuloma annulare

A

Granuloma annulare lesions consist of papules that form a ring that may mimic tinea corporis.
It may be differentiated from tinea corporis by the absence of scale, firmness of the border, and tendency of the lesions to have a violaceous color.
Granuloma annulare lesions resolve spontaneously within 2 to 4 years; no intervention is required.

222
Q

Describe Sinding-Larsen-Johansson syndrome (SLJS)

A

Distal patellar apophysitis, also known as Sinding-Larsen-Johansson syndrome (SLJS) is a traction apophysitis of the patellar tendon on the growth plate at the inferior pole of the patella. Atraumatic pain over the inferior pole of the patella in a skeletally immature child or adolescent is consistent with SLJS. Sinding-Larsen-Johansson syndrome is an overuse injury caused by repeated high-intensity activation of the extensor mechanism of the knee, such as running or jumping.

223
Q

Describe PHACE syndrome and LUMBAR syndrome

A

PHACE (posterior fossa anomalies, hemangioma on the face, arterial anomalies, cardiac anomalies, and eye anomalies) and LUMBAR (lower body IH and other cutaneous defects, urogenital anomalies and ulceration, myelopathy, bony deformities, anorectal malformations, and arterial anomalies, and renal anomalies) syndromes.

224
Q

When do you recommend liver US for Infantile hemangioma?

A

If patient has more than 5 lesions

225
Q

When Infantile hemangiomas should be early referred?

A

Early referral is indicated for IHs in certain locations:

Perioral: May interfere with feeding because of both the IH itself and its ulceration.

Airway: Usually present as worsening biphasic stridor and barky cough in the first 6 months after birth. These require early otolaryngology referral. It is important to note that half of all children with IHs involving the airway will have cutaneous IHs and approximately half of those who have cutaneous IHs in the “beard” distribution (Item C183J) will have an airway IH.

Orbital or periocular: Require early ophthalmology referral to mitigate and detect amblyopia, strabismus, and refractive errors because of the mechanical obstruction caused by the IH and, if the visual field is obstructed, the lack of stimulation of the visual cortex.

Nose (and other cosmetically significant areas): May require earlier surgical resection.

Perineum: Prone to ulceration. Surgical resection is complicated in this area; aggressive medical therapy is the treatment of choice.

Hepatic: Diffuse hepatic IHs, while rare, are extremely serious. Complications may include consumptive hypothyroidism because of type 3 iodothyronine deiodinase production, hepatomegaly, abdominal compartment syndrome, renal vein compression, and compromise of inferior vena cava flow. Rarely, large IHs or multiple hepatic IHs may result in high-output congestive heart failure because of shunting.

226
Q

Which are the most common organisms to cause infection of burn lesions?

A

The most common organisms that cause burn wound infections are Staphylococcus spp and Pseudomonas.

227
Q

Which syndrome is related to thrombocytopenia, eczema (atopic dermatitis), and infections ?

A

The triad of thrombocytopenia, eczema (atopic dermatitis), and infections suggest that he has Wiskott-Aldrich syndrome (WAS), an X-linked recessive disorder.

228
Q

Treatment for pseudotumor cerebri (increased intracranial pressure)?

A

The treatment of intracranial hypertension includes removing the causative agent(s), if present and medically appropriate to do so. Acetazolamide, the first-line medication used to lower ICP, decreases CSF production. Furosemide and topiramate can be used for those unable to tolerate acetazolamide. In severe or rapidly progressing cases, optic nerve fenestration or CSF diversion with shunt can be performed.

229
Q

How often are the wcc for kids in foster care?

A

The American Academy of Pediatrics recommends that regular visits for children in foster care occur monthly until age 6 months, every 3 months from age 6 to 24 months, and then every 6 months until age 21 years.

230
Q

Treatment for lichen sclerosis?

A

First-line treatment is usually a medium- to high-potency topical steroid. Many clinicians initiate therapy with an ultrapotent (group 1) agent (eg, clobetasol propionate or betamethasone dipropionate), tapering the frequency of application or potency as the condition improves. Once control is achieved, the topical steroid can be withdrawn, and maintenance therapy is initiated with a topical calcineurin inhibitor (eg, tacrolimus or pimecrolimus).

231
Q

When should we vaccinate kids with cancer?

A

Children who have cancer should be immunized with live vaccines when in remission, at least 3 months after chemotherapy, and with evidence of immunocompetence.

Inactivated-virus vaccines may be given to children with cancer who are receiving chemotherapy; however, unless there is documentation of a protective antibody level, inactivated vaccines given during chemotherapy should not be considered valid doses.

232
Q

What’s the treatment for Lyme arthritis?

A

The initial treatment for Lyme arthritis is oral antibiotics for 28 days, amoxicillin <8 years of age, doxycycline ≥8 years of age.

233
Q

Which are the most common causes of otorrhea?

A

The most common causes of otorrhea include otitis externa, foreign body in the external ear canal, and acute otitis media with tympanic membrane rupture.

234
Q

Target height for boys and girls?

A

The adjusted midparental height calculation for boys is:

(mother’s height in cm + 13 cm + father’s height in cm) divided by 2

Target height is the adjusted midparental height plus or minus 2 standard deviations.

The adjusted midparental height calculation for girls is:

(father’s height in cm - 13 cm + the mother’s height in cm) divided by 2

235
Q

Treatment for seborrheic dermatitis?

A

For skin affected by seborrheic dermatitis, treatment is with a low-potency topical corticosteroid (eg, hydrocortisone 1% or 2.5%) or an agent active against yeast (eg, clotrimazole, miconazole nitrate, or ketoconazole) applied twice daily as needed. Scalp involvement is treated with an antiseborrheic shampoo containing pyrithione zinc, selenium sulfide, or ketoconazole. If the scalp and face are involved, some shampoo should be allowed to contact affected areas and then rinsed off. If signs of scalp inflammation are present (eg, erythema or erosions), a topical corticosteroid solution (eg, fluocinolone acetonide 0.1%) may be applied at bedtime.

236
Q

Describe simple febrile seizure

A

for a febrile seizure to be designated as simple, it must meet all of these 5 criteria:

Occur in a child between 6 months through 5 years of age
Occur in a developmentally normal child
Be generalized with no focal features
Last less than 15 minutes
Occur once in a 24-hour period

237
Q

Indications to treat giardiasis?

A

Indications for treatment include acute or chronic diarrhea associated with failure to thrive, malabsorption syndrome, extraintestinal disease, or infection in immunocompromised hosts.

238
Q

Treatment options for giardiasis?

A

Oral antiparasitic treatment options are dependent on age. These include: tinidazole (single dose) for children aged >3 years, nitazoxanide (3-day course) for children aged 1 year or older; and metronidazole (5- to 7-day course).

239
Q

main findings of congenital cmv infection ?

A

Congenital cytomegalovirus infection causes significant injury to the developing central nervous system,
leading to microcephaly and periventricular calcifications. Chorioretinitis, hepatosplenomegaly, jaundice,
intrauterine growth restriction and a blueberry muffin rash may also be encountered. A progressive
sensorineural hearing loss makes congenital CMV the most common cause of non-inherited deafness.

240
Q

main findings of congenital rubella infection?

A

Microcephaly, congenital cataracts, and a patent ductus arteriosus (characterized by a continuous harsh
“machinery-like” murmur associated with bounding pulses and often a thrill at the 2nd intercostal space)
are typical findings of congenital rubella syndrome. Additional cardiac findings may include peripheral
pulmonary artery stenosis and valvular pulmonic stenosis. Sensorineural hearing loss often accompanies
significant developmental delay and behavioral problems. A purpuric, generalized “blueberry muffin”
rash is another characteristic of congenital rubella.

241
Q

What are the Symptoms and signs suggestive of inborn errors of metabolism?

A

Symptoms and signs suggestive of inborn errors of metabolism include developmental regression, abnormal tone, ataxia, seizures, organomegaly, atypical odor, and failure to thrive.

242
Q

Diagnosis of HLH?

A

Five of the following 8 criteria:
Fever
Splenomegaly
Cytopenias in 2 out of the 3 blood cell lines
Hypertriglyceridemia and/or hypofibrinogenemia
Hemophagocytosis in the spleen, bone marrow, or lymph node without evidence of malignancy
Low or absent natural killer (NK) cell activity
Elevated ferritin level
Elevated soluble CD25 (IL-2 receptor) level

243
Q

whats the Investigation for HLH after the diagnosis is stablished?

A

Once a child is diagnosed with HLH, additional investigations must be performed including:

Genetic testing for primary HLH
Evaluation for infectious causes of secondary HLH
Bone marrow aspirate and biopsy to rule out malignancy and look for the classic hemophagocytosis macrophage
Lumbar puncture (to determine if intrathecal therapy is indicated)
Brain imaging to assess for central nervous system disease

244
Q

Which are the common acute complications of bacterial meningitis?

A

Common acute complications of bacterial meningitis include increased intracranial pressure, hydrocephalus, seizures, stroke, hemorrhage, and subdural empyemas.

245
Q

Which are the long-term complications from bacterial meningitis?

A

Long-term neurological sequalae of bacterial meningitis are common, especially pneumococcal meningitis; these include spastic paresis, seizures, cognitive impairment, and sensorineural hearing loss.

246
Q

What are the findings of salicylate toxicity?

A

Findings of salicylate toxicity may include nausea, vomiting, abdominal pain, changes in hearing, tinnitus, diaphoresis, tachypnea, tachycardia, and altered mental status coupled with a mixed respiratory alkalosis and metabolic acidosis on serum blood gas analysis.

247
Q

For what the tzanck smear is used for?

A

Tzanck is used to identify herpetic eruptions, which present with pink vesicles or crusted erosions (varicella), or grouped vesicles on an erythematous base (herpes simplex virus).

248
Q

What is lichen striatus?

A

lichen striatus, an asymptomatic or mildly pruritic, self-limited inflammatory disorder that typically resolves over the course of several months. Due to the self-limited nature, treatment is not indicated. Topical corticosteroids may help with any associated pruritus but will not hasten resolution. It presents with grouped 2–5 mm pink to hypopigmented, flat-topped papules in a curvilenear, blaschkoid distribution (following embryonic ectodermal lines of migration), most often located on an extremity. Darker skin types tend to have more hypopigmented presentations.

249
Q

What is PANDAS syndrome?

A

PANDAS syndrome means pediatric autoimmune neuropsychiatric disorder associated with group A streptococci

250
Q

Describe VACTERL

A

VACTERL association is a disorder that can include vertebral, anal atresia, cardiac, tracheal, esophageal, renal, and limb defects. Patients diagnosed with VACTERL association typically have at least three of these characteristic features and may have additional abnormalities.

251
Q

describe Fanconi Anemia

A

Fanconi anemia is associated with progressive bone marrow failure that manifests as thrombocytopenia, leukopenia, and anemia (pancytopenia). Presentation is typically in the first decade of life, often initially with either thrombocytopenia or neutropenia. Bone marrow, initially normocellular, becomes progressively hypoplastic with time. Birth defects occur in up to 75% of people with Fanconi anemia and include gastrointestinal atresias and radial abnormalities (including absent thumbs). Many will have generalized skin hyperpigmentation or café au lait macules.

252
Q

Describe Diamond-Blackfan anemia

A

Diamond-Blackfan anemia (a.k.a. congenital hypoplastic anemia) is associated with a profound isolated anemia with a median age of presentation at 2 months of age. Birth defects occur in about 50% of affected individuals and include microcephaly, cleft lip and/or palate, Klippel-Feil anomaly, triphalangeal or absent thumb, renal defects, and heart defects.

253
Q

Describe BPVC

A

benign paroxysmal vertigo of childhood (BPVC) occurs in children 1–4 years of age with sudden onset, recurrent vertigo. This can cause children to have ataxia, nystagmus (the patient’s “abnormal eye movements” after she moves), pallor, and emesis. Children often do not want to move at all. Unlike with a focal seizure, a key distinguishing feature of BPVC is retained consciousness during the episode with a lack of postictal state and reproducible signs/symptoms during each attack (e.g., nystagmus and worsening vertigo with head movement). BPVC is a precursor to migraines. Children with BPVC often have a strong family history of migraine and go on to develop migraines themselves.

254
Q

Most common cause of intoeing in a child older than 2yo?

A

The most common cause of intoeing in a child older than 2 years is internal femoral torsion (a.k.a. femoral anteversion). In the prone position, there is increased internal rotation of the hip, greater than 45°. External rotation is generally less than 45°. Generalized ligamentous laxity may be present.

Radiographic evaluation is not necessary. Most of the time this self-corrects and no intervention is necessary. If the deformity persists beyond 11 years of age, has poor cosmetic appearance, interferes with activity, or if the condition is unilateral, consult an orthopedist with experience in rotational deformities.

255
Q

What is the most common cause of cerebral palsy?

A

Prematurity is the most common risk factor for the development of CP (35%). Although some of this risk relates to complications of prematurity (e.g., increased risk of intraventricular hemorrhage), low birth weight remains the highest risk factor for the development of CP. In term babies, intrapartum asphyxia and infection are major causes of CP.

256
Q

Describe what is a ranula

A

Ranula is a pseudocyst associated with the sublingual glands and/or submandibular ducts. They are characterized by a translucent to bluish, nonblanching, fluctuant swelling lateral to the midline of the lower mouth. Its appearance is sometimes compared to the belly of a frog.

257
Q

What is the treatment for Diamond-Blackfan anemia?

A

Transfusion therapy, corticosteroids, and stem cell transplantation are treatment options. Transfusion therapy is the treatment of choice in the first 12 months of life to avoid negative side effects on bone growth. A trial of corticosteroid therapy is recommended after 12 months of age; up to 80% of patients respond to corticosteroid therapy within 2–4 weeks, but treatment must often be discontinued because of associated side effects. Patients who do not respond to or must discontinue corticosteroid therapy require frequent transfusions (every 3–6 weeks) and eventual stem cell transplantation.

258
Q

What do you find in a peripheral smear from a patient with Diamond Blackfan Anemia?

A

On peripheral smear in patients with Diamond-Blackfan anemia, the red blood cells are normochromic and macrocytic without associated hypersegmentation of neutrophils

259
Q

What is the Galleazi sign and what does It diagnoses?

A

The Galeazzi sign initially presents at 8–10 weeks of age and is defined by asymmetry in the height of the knees due to shortening of the thigh on the affected side. It is best observed after placing both hips in 90° flexion. Galeazzi sign is more helpful than other maneuvers in detecting DDH in children > 3 months of age. In addition to the Galeazzi sign, clinical findings in infants older than 8–10 weeks of age include limited hip abduction, asymmetry of the gluteal and thigh folds, and “pistoning” of the hip.

260
Q

Most common complication associated with sickle cell trait?

A

renal papillary necrosis with gross hematuria

261
Q

Describe Potter syndrome

A

Potter sequence is characterized by dysmorphic facial features (flattened nose, recessed chin, prominent epicanthal folds, and low-set abnormal ears) and limb anomalies (altered positioning of the hands and feet, club feet, and/or dislocation of the hips) caused by intrauterine compression and limitation of fetal movement due to oligohydramnios. In this syndrome, oligohydramnios results from decreased urine output associated with bilateral renal agenesis, cystic renal dysplasia, obstructive uropathy, or other underlying disorders of the kidneys. Other causes of oligohydramnios, such as chronic leakage of amniotic fluid, may be associated with Potter sequence.

262
Q

Which are the best options to treat head lice? And when should the child return to school after treatment?

A

The child can return to school the following day after initiating treatment with ivermectin 0.5% lotion. Ivermectin is approved for ages 6 months and up for the treatment of head lice. It is a good option because it lacks neurotoxic effects and the over-the-counter alternative permethrin 1% has resistance rates as high as 50% in some areas of the US. Benzoyl alcohol 5% lotion is another treatment option that is approved, lacks the neurotoxic effects, and is an alternative to the pesticides.

263
Q

What happens on Hypophosphatemic rickets and how do you treat it?

A

Hypophosphatemic rickets is due to decreased renal resorption of phosphorus at the renal tubule. This is caused by increased FGF23 effect in the kidney, which leads to phosphorus wasting and decreased 1-alpha hydroxylase activity. While these patients’ 25-hydroxy vitamin D levels are normal, 1,25-dihydroxy vitamin D is inappropriately normal considering the degree of hypophosphatemia. Treatment for hypophosphatemic rickets consists of phosphorus supplementation and calcitriol therapy. The calcitriol is used to maximize the reabsorption of phosphorus. The most common form is X-linked dominant. X-linked recessive, autosomal dominant, and autosomal recessive forms are rare.

264
Q

Describe reactive arthritis

A

The patient has clinical and laboratory findings consistent with reactive arthritis, an autoimmune reaction triggered by a recent (within 2–4 weeks) enteric or genitourinary (sexually transmitted) infection. One of several spondyloarthropathies, reactive arthritis classic findings include the triad of conjunctivitis, arthritis, and urethritis. Oligoarthritis is associated with focal tenderness at sites where ligament and tendon insert into bone (enthesitis). Circinate (gyrate) balanitis (inflamed, hyperkeratotic, whitish plaques on the glans penis) is common in patients with reactive arthritis. In uncircumcised males, it is characterized by well-defined erosions; and in circumcised males, it is characterized by inflamed hyperkeratotic whitish plaques that spread centrifugally, eventually covering most or all of the glans penis. Lesions may also involve the penile shaft and/or scrotum. Circinate vulvitis may occur in female patients. Unlike patients with ankylosing spondylitis, reactive arthritis is not associated with morning stiffness and low back pain. Patients with ankylosing spondylitis do not have conjunctivitis or urethritis.

265
Q

For how long can a baby develop perinatal HPV warts?

A

Perinatally acquired condyloma acuminata can present in children up to 20 months of age.

266
Q

Describe Menkes disease

A

Menkes disease (a.k.a. Menkes kinky hair disease) is an X-linked recessive dysfunction of copper metabolism with impaired absorption and transport. It is characterized by marked intellectual disabilities and collagen abnormalities resulting in skeletal and vascular pathology and abnormal hair. Copper levels are suppressed.

267
Q

What do you need to consider before testing kids <1 year of age to clostridium difficile?

A

Be wary of ordering C. difficile tests in young children, especially those < 1 year of age, as up to 50% of healthy infants are colonized with C. difficile (which may not be the actual cause of the diarrhea).

268
Q

Describe an asthma action plan for a 12yo (mild exacerbation)

A

The initial treatment of patients with a mild asthma exacerbation at home is with either 2–4 puffs of a short acting β2-agonist (SABA) or 1–2 puffs of inhaled corticosteroids (ICS) along with 2 puffs of a SABA.

Patients should be reassessed in 10–20 minutes and can receive 2 additional puffs of SABA if wheezing, tachypnea, or shortness of breath persists. Home management should always feature an asthma action plan, along with instructions on management of acute exacerbations, including medications and how and when to contact a clinician.

Oral glucocorticoids (2 mg/kg/day for 5 days) can be started at home in patients who have an incomplete response to 2 doses of SABA. An incomplete response is defined as persistent wheezing, tachypnea, and shortness of breath.

269
Q

Side effects of SSRI late in pregnancy?

A

Neonates exposed to SSRIs late in the third trimester may present with respiratory disturbances, somnolence/ hypotonia, jitteriness, increased muscle tone, hypoglycemia, vomiting and diarrhea, feeding disturbances, and (rarely) seizures. These effects are noted most commonly either at birth or within a few days of birth. Most infants require only symptomatic care in a normal newborn nursery setting and symptoms usually resolve within 1–2 weeks.
The use of SSRIs, particularly in late pregnancy, may also increase the risk of persistent pulmonary hypertension in the newborn.

270
Q

What’s the treatment for Henoch Shcollein purport (IgA vasculitis)?

A

Treatment is symptomatic and includes rest, fluids, and NSAIDs for pain control.

271
Q

Which condition is related to Alice in wonderland syndrome?

A

Perceptual distortions may be a presenting symptom of infectious mononucleosis. Patients may complain of distorted shapes, sizes, colors, and/or spatial relationships. This disorienting neurologic condition is sometimes referred to as “Alice in Wonderland syndrome.” Additional neurologic complications of infectious mononucleosis include seizures, ataxia, meningitis, transverse myelitis, encephalitis, and facial nerve palsies.

Alice in Wonderland syndrome may also be caused by migrainous ischemia and cortical irritability. It has been associated with viral encephalitis, epilepsy, and head trauma.

272
Q

Which type of polydactyly is more common pre or post axial?

A

Postaxial (ulnar or 5th finger) polydactyly is more common than preaxial (radial or thumb) polydactyly.

273
Q

Which type of polydactyly is more related to genetic syndrome?

A

Preaxial duplications (in contrast to postaxial) are often associated with a syndrome.

274
Q

Mention the branched chain amino acids

A

Branched-chain amino acids include leucine, isoleucine, and valine.

275
Q

What is the clinical difference between turner and Noonan syndrome?

A

Noonan syndrome and Turner syndrome share many clinical features. These include short stature; short, webbed neck; a low-set posterior hairline; a shieldlike chest with widely spaced nipples; pectus excavatum; epicanthal folds; and increased carrying angle of the arms. A different pattern of congenital heart disease describes differences between Noonan syndrome and Turner syndrome.

276
Q

Describe common findings in Noonan syndrome

A

Patients with Noonan syndrome have related findings, which include delayed puberty, male cryptorchidism associated with a small penis, and bleeding disorders. Unlike Turner syndrome (45,X), which only affects girls, Noonan syndrome may occur in either sex and is associated with normal chromosomes (it is a single gene disorder rather than a chromosomal abnormality). Patients with Noonan syndrome may have pulmonic stenosis (a.k.a. pulmonary stenosis) often associated with a dysplastic pulmonary valve. Atrial septal defects, tetralogy of Fallot, and hypertrophic cardiomyopathy are also more frequent in children with Noonan syndrome.

277
Q

Which are the indications to treat PID in the hospital setting?

A

Indications for hospitalization for women with PID include severe illness, pelvic abscess, need for surgical intervention, lack of response or inability to tolerate outpatient treatment, and risk of noncompliance with outpatient therapy.

278
Q

What is the latest manifestation of rheumatic fever?

A

Chorea

279
Q

Whats the treatment of choice for clostridium difficile?

A

Oral metronidazole or oral vancomycin are recommended for initial treatment of children and adolescents with mild-to-moderate disease and for 1st recurrence. For severe disease, characterized by hypotension and severe abdominal pain, oral vancomycin is preferred (with or without IV metronidazole).

280
Q

Which one is the most common soft tissue tumor from childhood?

A

Rhabdomyosarcoma

It can arise in any muscle in the body but is most common in the head and neck area or in the pelvis. X-rays are nonspecific, and a tissue biopsy is required for an accurate diagnosis.

281
Q

Describe the findings for osteosarcoma

A

Osteosarcomas are rare primary malignant tumors of bone. Typically, the patient presents with localized pain, frequently of several months’ duration, and a soft tissue mass that is usually large and tender. On x-rays, there is a mixture of radiodense and radiolucent areas, with periosteal new bone formation, lifting of the cortex, and formation of a “Codman triangle.” The associated soft tissue mass is variably ossified in a radial or sunburst pattern.

282
Q

Describe osteoid osteoma

A

Osteoid osteoma is a benign bone-forming tumor characterized by a small radiolucent nidus (usually < 1 cm to 1.5 cm in diameter). The patients usually present with insidious pain that is worse at night relieved by nonsteroidal antiinflammatory medication and usually unrelated to activity.

283
Q

Which symptoms are considered high risk for depression?

A
  • Family history of depression or bipolar disorder
  • Suicide-related behaviors
  • Substance use
  • Other psychiatric illness
  • Significant psychosocial stressors, such as family crises, physical and sexual abuse, neglect, and other trauma history
  • Frequent somatic complaints
  • Foster care and adoption
  • Personal history of previous depressive episodes
  • Other psychiatric disorders
  • Previous high-scoring screen without a diagnosis
284
Q

At which age do you find physiologic genome valgus and varus?

A

Birth—2 years: varus (bowing) of the knees
2—4 years: valgus (knock-knee) posture
4—7 years: levels off to 4–7° valgus in males and 5–9° in females

285
Q

Which eye finding is commonly seeing in chronic barbiturate use?

A

nystagmus

286
Q

Which findings do you see in overdose for barbiturates?

A

Overdose of barbiturates is associated with respiratory depression, severe orthostatic changes in blood pressure, and slowing of the heart rate.

287
Q

Which drug tests can be false positive after ingesting ibuprofen?

A

Ibuprofen use can cause positive screening for PCP, cannabinoids, barbiturates, and/or benzodiazepines.

288
Q

Which drug test can be false positive after ingesting proton pump inhibitor ?

A

Proton pump inhibitors can cause false-positive results for Cannabis.

289
Q

Side effects of risperidone?

A

Adverse effects include liver function abnormalities, dyslipidemia, insulin resistance leading to elevation of blood sugar, increase in blood pressure, and electrocardiographic changes, including prolongation of QTc interval. Although the clinical significance is unknown, many patients also have increased serum levels of prolactin. Other side effects include generalized fatigue, lethargy, weight gain with increase in appetite, and drooling.

290
Q

What’s the complication of a not repaired TOF?

A

Brain abscesses are a well-known complication of unrepaired TOF in patients who have a significant
right to left shunt, along with cerebral thrombosis with hemiplegia and infective endocarditis.

291
Q

How often should we screen celiac disease in turner syndrome?

A

Screening for celiac disease is recommended in Turner syndrome every 2–5 years during childhood and as needed after childhood.

292
Q

What do you see on the peripheral smear from HUS ?

A

burr cells, schistocytes, red blood cell fragments that look like helmets

293
Q

Metabolic changes in patient with bulimia?

A

Patients with recurrent vomiting associated with bulimia nervosa (BN) or binge eating/purging-type of anorexia nervosa (AN) may suffer from hypokalemia and hypochloremic metabolic alkalosis resulting from loss of gastric hydrochloric acid, chronic dehydration, and the subsequent increase in aldosterone that promotes sodium reabsorption in exchange for potassium and acid at the distal tubule level.

294
Q

Describe subcutaneous fat necrosis of newborn

A

Subcutaneous fat necrosis (SCFN) of the newborn is a rare, self-limited panniculitis that typically occurs in full-term or post-term newborn infants within the first few weeks of life. SCFN is clinically characterized by firm, red or purple subcutaneous nodules and plaques. SCFN usually has a favorable outcome with spontaneous resolution over several weeks or months. Nonetheless, it may be complicated by critical metabolic alterations, principally hypercalcemia, which is associated with significant morbidity and mortality.

295
Q

How to monitor babies with subcutaneous fat necrosis of newborn?

A

Timely identification of this condition is critical as it can additionally display significant extracutaneous abnormalities, including thrombocytopenia, hypertriglyceridemia, and hypercalcemia. For the most definitive diagnosis, both clinical presentation and skin biopsy must be considered.[6] However, to avoid skin biopsy, ultrasonography examination, along with Doppler blood flow analysis, has been proven to be effective in detecting subcutaneous fat necrosis of the newborn, it typically indicates subcutaneous high echo signal with or without calcifications.

Del Pozzo-Magaña and Ho suggested guidelines for monitoring patients with SCFN. Once SCFN is diagnosed, serum levels of triglycerides, renal function tests, platelets, and glucose should be determined. In patients with normal values, only clinical follow-up is required. It is imperative to measure serum levels of ionized calcium in normocalcemic patients, it should be repeated weekly during the first month of life and then monthly until six months of age or after the resolution of skin lesions. Also, it is crucial to perform a renal ultrasound, especially in patients with hypercalcemia. When the initial imaging is normal, it can be repeated at three months of age or after the resolution of skin lesions.

296
Q

First line treatment for infantile spasms?

A

Adrenocorticotropic hormone (ACTH; a.k.a. corticotropin), oral prednisolone, and oral vigabatrin are considered 1st line treatments for infantile spasms.

297
Q

Describe Dandy Walker malformation

A

Dandy-Walker malformation is a congenital malformation of the cerebellum, including enlargement of the 4th ventricle and absence of the cerebellar vermis. Patients rarely are symptomatic; when they are, they may experience headache, gait disturbance, or develop hydrocephalus.

298
Q

Describe Chiari Malformation

A

The presentation of Chiari malformation varies in the severity of the tonsillar herniation and associated other features. Chiari malformation Type I involves descent of tonsils into the foramen magnum. Arnold-Chiari malformation (a.k.a. Chiari malformation Type II) involves descent of the cerebellar vermis, 4th ventricle, and medulla into the foramen magnum. Depending on the degree of descent, the hydrodynamics of the cerebrospinal fluid (CSF) flow can be partially obstructed, and this is a contributor to symptoms. Suboccipital headaches are common. Patients with Chiari malformations need spinal imaging to rule out spinal cord disorders, such as syringomyelia in Chiari malformation Type I or myelomeningocele in Chiari malformation Type II.

299
Q

Describe Crouzon syndrome

A

It results from bilateral, premature closure of the coronal sutures (brachycephaly). Patients have midface hypoplasia, proptosis, normal intelligence, and normal hands and feet. 1/3 of patients have associated hydrocephalus.

300
Q

Which are the classical features for mucopolissacaridosis?

A

The classic features of an MPS include coarse facial features, frequent upper respiratory infections, macrocephaly, hernias, and thickened heart valves.

301
Q

Which are the laboratory findings in rhabdomyiolysis ?

A

hyperphosphatemia, hypocalcemia, hyperuricemia, hyperkalemia and hypoalbuminemia.

The WBC count is often elevated in rhabdomyolysis, as are serum transaminase levels and renal function tests.

302
Q

what’s the triad of mccune Albright syndrome?

A

MAS is typically composed of a triad of precocious isosexual puberty, café au lait skin lesions, and fibrous dysplasia of the bones, often on 1 side of the body.

303
Q

How much vitamin D should an infant receive daily?

A

The American Academy of Pediatrics recommends that breastfed infants receive at least 400 IU of supplemental vitamin D daily.

304
Q

A diffuse petechial rash that involves the palms and soles is suggestive of…

A

Rocky Mountain spotted fever

305
Q

What are the diagnostic criteria for Toxic shock syndrome?

A

Clinical diagnostic criteria for S. aureus TSS include fever ≥ 102.0° F (38.9° C); diffuse, macular, sunburn-like erythroderma (followed by desquamation in 7–14 days); and hypotension. Also included is the involvement of ≥ 3 organ systems:

Gastrointestinal (vomiting and diarrhea)
Hepatic (total bilirubin or transaminase levels > 2x normal)
Muscular (myalgia or creatine kinase level at least 2x normal)
Renal (BUN, creatinine levels at least 2x normal or > 5 leukocytes/HPF on urine microscopy)
Central nervous system (altered mental status without focal neurologic findings)
Hematologic (platelet count ≤ 100,000/mm3; mucous membrane hyperemia)

306
Q

Main causes of orchitis by age group?

A

Children ages 2 to 13 years often have a postinfectious cause (most commonly Mycoplasma, enterovirus, or adenovirus). Vasculitis (eg, Henoch-Schönlein purpura) is another common cause of orchitis and epididymitis in this age group. Older children and adults are more likely to have an infectious etiology with organisms that cause urinary tract or sexually transmitted infections.

307
Q

Lab findings on pyloric stenosis?

A

hypochloremic, hypokalemic metabolic alkalosis.

308
Q

Diagnostic criteria for Tourette Syndrome?

A

At least 2 motor tics and at least 1 vocal (phonic) tic have been present, not necessarily at the same time.

Tics may wax and wane in frequency but have occurred for more than 1 year.

Tics started to appear before the age of 18.

Tics are not caused by the use of a substance or other medical condition.

309
Q

What is postpericardiotomy syndrome?

A

postpericardiotomy syndrome, characterized by the development of pericarditis and/or pericardial effusion days to several weeks after cardiac surgery.

310
Q

What’s the recommendation for tonsillectomy after GAS infection?

A

Referral for tonsillectomy is indicated in patients with 7 episodes in the past year, 5 in each of the past 2 years, or 3 in each of the past 3 years.

311
Q

What’s the difference between Osgood-Schlatter disease and Sinding-Larsen-Johansson syndrome ?

A

Young people diagnosed with SLJ injure the tendon right where it leaves the kneecap, while those with Osgood-Schlatter disease damage the tendon at its opposite end, where it exits the shin. It can be hard to tell the difference between Sinding-Larsen-Johansson syndrome and Osgood-Schlatter disease.

312
Q

Diagnostic criteria of Acute otitis media?

A

The diagnostic criteria for AOM include:

Recent onset of ear pain (<48 hours)
Bulging tympanic membrane
Reduced mobility of the tympanic membrane with pneumatic otoscopy or tympanometry

Severe disease is defined as one of the following:

Otalgia that is moderate or severe in nature lasting for at least 48 hours
Temperature of 39°C (102.2°F) or higher

313
Q

How long is the Treatment for AOM?

A

the best treatment for children without severe signs or symptoms is observation for 48 to 72 hours with analgesic administration as needed followed by an antibiotic prescription if symptoms worsen or persist over that time frame.

Recommended duration of oral antibiotic treatment of AOM by age:

<2 years 10 days
2-5 years 7 days
≥6 years 5 to 7 days

314
Q

What is the age/ height to outgrow the booster seat?

A

Children should stay in a booster seat until adult belts fit correctly (usually when a child reaches about 4’ 9” in height and is between 8 and 12 years of age.

315
Q

CBC in Diamond-Blackfan anemia

A

This diagnosis is supported by severe macrocytic anemia without reticulocytosis, elevated fetal hemoglobin level, and elevated erythrocyte adenosine deaminase (eADA) level.

316
Q

For which substances is fomepizole used as antidote?

A

Fomepizole is the antidote for ethylene glycol and methanol toxicity

317
Q

Restless leg syndrome is associated with…

A

Low levels of ferritin

318
Q

how much formula should an infant receive to have adequate amount of vitamin D?

A

Formula-fed neonates must consume 32 ounces daily to receive the recommended amount of vitamin D.

319
Q

Iron supplementation after 4mo?

A

The AAP recommends iron supplementation (1 mg/kg/day) for all exclusively breastfed infants until the intake of iron-containing foods (eg, iron-fortified cereal, meats, lentils) is sufficient to meet requirements. For partially breastfed infants, the intake of iron through formula and other dietary sources should be considered prior to supplementation. As most infant formulas contain 10-12 mg/dL iron, formula-fed infants do not need supplementation. The recommended dietary iron intake is 11 mg/day for infants aged 6 to 12 months.

320
Q

Risk factors for neonatal sepsis?

A

risk for sepsis due to a combination of factors, late preterm gestation, maternal fever, and prolonged rupture of membranes (PROM, >18 hours)

321
Q

What are the indications to start antibiotics for risk of neonatal sepsis?

A

A suspected intra-amniotic infection is defined by the American College of Obstetricians and Gynecologists as maternal fever (>39°C) and at least 1 of the following: maternal leukocytosis, purulent cervical discharge, and fetal tachycardia. If the mother meets the criteria for suspected intra-amniotic infection, laboratory evaluation and empiric treatment with antibiotics are recommended for neonates with any of the following:

Signs of sepsis (eg, ill appearing)
Gestational age <37 weeks
Maternal PROM
Inadequately treated maternal GBS colonization
Evidence of maternal chorioamnionitis

322
Q

Describe Wiskott-Aldrich syndrome (WAS)

A

WAS is an X-linked disease which frequently presents with the classic triad of thrombocytopenia, eczema, and susceptibility to encapsulated bacteria and opportunistic infections.

323
Q

which are the complications to the fetus if carbamazepine is taken during pregnancy?

A

spina bifida, craniofacial defects, cardiovascular malformations, and hypospadias.

324
Q

side effects of isotretinoin if taken during pregnancy

A

increased risk for congenital defects in infants exposed to the drug in utero, including craniofacial, cardiovascular, neurological, and thymic malformations. About 30–60% of children exposed to isotretinoin prenatally have been reported to show neurocognitive impairment, even in the absence of physical defects.

325
Q

which drug is related to gastroschisis if taken during pregnancy?

A

cocaine

326
Q

genetic conditions associated with natal teeth

A

Natal teeth are rarely associated with genetic syndromes, particularly chondroectodermal dysplasia (a.k.a. Ellis-van Creveld syndrome), oculomandibulodyscephaly with hypotrichosis (a.k.a. Hallermann-Streiff syndrome), and pachyonychia congenita (a.k.a. Jadassohn-Lewandowski syndrome).

327
Q

What is the clinical presentation from T1 injury?

A

anisocoria

328
Q

Which findings can we see in patients with crasniosinostosis?

A

Affected individuals have asymmetric facies, midfacial hypoplasia, low-set ears, ocular proptosis, and hypertelorism.

329
Q

Describe craniotabes

A

The patient has findings typical of craniotabes, characterized by a softened, thinned, flexible area of the cranial bones, usually over the parietal or occipital region. Depression of the area gives the sensation of pressing—and then releasing—a “ping-pong ball” effect. Craniotabes may be one of the physical manifestations of syphilis, which is caused by Treponema pallidum.

330
Q

What’s the indication to repair an umbilical hernia?

A

surgery is usually deferred unless the patient becomes symptomatic, the defect increases in size between 1 and 2 years of age, or the hernia is still present at 5–6 years of age.

331
Q

Apert syndrome?

A

Apert syndrome, an autosomal dominant disorder characterized by brachycephaly, a high cranium, wide-set eyes, midface hypoplasia, choanal stenosis, nasopharyngeal airway obstruction, and symmetrical extremity syndactyly.

332
Q

Which drugs are used in night clubs to sedate people?

A

“club drugs” are a group of psychoactive compounds that are often used by adolescents and young adults at nightclubs, bars, and “rave dances” or “raves.” Examples include gamma-hydroxybutyrate (GHB), flunitrazepam, ketamine, 3,4-methylenedioxymethamphetamine (MDMA), and methamphetamine.

333
Q

First sign to be found in Rest syndrome?

A

Deceleration of head growth is often the 1st clinical sign and can occur as early as 3 months of age

334
Q

Ibuprofen can cause false positive in which drugs on the top screen?

A

Ibuprofen use can cause positive screening for PCP, cannabinoids, barbiturates, and/or benzodiazepines. Proton pump inhibitors can cause false-positive results for Cannabis.

335
Q

Symptoms of opioid withdrawal

A

opioid withdrawal are yawning, restlessness, dysphoria, rhinorrhea, lacrimation, mydriasis, diarrhea, myalgia/arthralgia, tachycardia, and systolic hypertension.

336
Q

CDC criteria for EVALI

A

E-cigarette use within the 90 days prior to symptom onset
Bilateral ground-glass opacities on chest x-ray and/or chest computed tomography (CT)
Evaluation indicating no other primary cause of disease

337
Q

Which drug causes Dissociation and loss of time perception

A

ketamine

338
Q

Ataxia, diplopia, lateral nystagmus, and euphoria describe adverse effects of…

A

barbiturate

339
Q

Orthopedic side effect of androgenic steroids?

A

Orthopedic complications include acceleration of maturation with early epiphyseal closure and decrease in ultimate adult height.e

340
Q

early signs of autism?

A

early signs of autism include absence of babbling, pointing, or other gestures by 14 months of age; not using single words by 16 months of age, or 2-word phrases by 24 months of age; lack of make-believe play by 18 months of age.

341
Q

language at 18, 24, 30, 36mo and 4yo?

A

An 18-month-old child uses > 10–25 words, follows simple commands, and can identify 2–4 body parts.

A 2-year-old child can use 2-word utterances, which are at least 50% intelligible. He can identify 6 body parts, use personal pronouns, and follow a 2-step command.

A 30-month-old child uses pronouns correctly, understands prepositions, refers to self with the correct pronoun, and can recite parts of well-known songs or stories.

A 3-year-old child uses phrases of 3 or more words, 75% of which are intelligible. He can identify 2–3 colors, use plurals, and ask “who” and “what” questions.

A 4-year-old child uses phrases of 4 or more words, which are 100% intelligible to strangers. He can identify 5–6 colors, use past and present tenses, and ask “why” questions.

342
Q

Crouzon syndrome?

A

Crouzon syndrome is the most common of these syndromes. It results from bilateral, premature closure of the coronal sutures (brachycephaly). Patients have midface hypoplasia, proptosis, normal intelligence, and normal hands and feet. 1/3 of patients have associated hydrocephalus.

343
Q

Apert syndrome?

A

Apert syndrome involves the premature closure of multiple sutures (though often bicoronal synostosis is seen, as in Crouzon syndrome). These patients have syndactyly of the hands and feet with the characteristic mitten hand, as well as an intellectual disability.

344
Q

Carpenter syndrome?

A

Carpenter syndrome includes the fusion of multiple sutures (commonly lambdoid and sagittal sutures), syndactyly, intellectual disability, and, less frequently, CHD, orthopedic abnormalities, and corneal opacities.

345
Q

Pfeifer syndrome?

A

Pfeiffer syndrome consists of brachycephaly (in Type 1) and a cloverleaf skull (in Type 2), midface hypoplasia, and finger/toe abnormalities. Type 3 is similar to type 2; however, the cloverleaf skull deformity is absent. Type 1 is generally mild, but Types 2 and 3 are associated with severe neurologic deficits.

346
Q

At what age can a child Draw a circle?

A

3 years of age

347
Q

Neonatal behavioral syndrome?

A

Antenatal exposure to SSRIs in late pregnancy may result in neonatal behavioral syndrome (poor neonatal adaptation), characterized by hypothermia, vomiting, frequent stools, feeding difficulties, poor sleep, excessive crying, agitation, irritability, tremors, jitteriness, respiratory problems, hypoglycemia, seizures, hypertonia, and hyperreflexia.

348
Q

Most common suture involved in craniosynostosis

A

The most common type of craniosynostosis is premature fusion of the midline sagittal suture known as scaphocephaly (a.k.a. dolichocephaly)

349
Q
A
350
Q

Example of anticholinergic meds ?

A

Atropine, antihistamines and tricyclic antidepressant

351
Q

Which poisoning gives prolonged QRS?

A

Tryciclic poisoning

352
Q

Which poisonings gives elevated serum os mil gap?

A

Methanol, ethylene glycol, isopropyl alcohol

353
Q

Which poisoning gives high anion gap?

A

MUDPILES
Methanol, uremia, DKA, propylene glycol, iron/isoniazid, lactate, ethanol/ethylene glycol, salicylates

Cyanide, carbon monoxide

354
Q

For which substances is Activated charcoal is in effective or contraindicated ?

A

CHEMICAL CAMP
Caustics
Hydrocarbon (most water soluble compounds)
Electrolytes
Metals
Iron
Cyanide
Alcohols
Lithium
Camphor
Phosphorus

355
Q

What is the minimal toxic dose of acetaminophen in a child ?

A

< 12 y: 150mg/kg
Teenagers/adults: 7.5g

356
Q

Ferric chloride turns urine purple or brown if……. Are present

A

Salicylates